Site Loader

Содержание

Принцип ферма и законы геометрической оптики. Роль принципа ферма в оптике. Доказательство закона преломления света с помощью принципа Ферма

Доказательство
закона отражения света
из принципа Ферма

Принцип Ферма (принцип наименьшего времени Ферма) в геометрической оптике — постулат, предписывающий лучу света двигаться из начальной точки в конечную точку по пути, минимизирующему (реже — максимизирующему) время движения (или, что то же самое, минимизирующему оптическую длину пути).

Этот принцип, сформулированный в I в. Героном Александрийским для отражения света, в общем виде был сформулирован Пьером Ферма около 1660 года в качестве самого общего закона геометрической оптики. В разнообразных конкретных случаях из него следовали уже известные законы: прямолинейность луча света в однородной среде, законы отражения и преломления света на границе двух прозрачных сред .

Законы геометрической оптики и принцип Ферма

Конечность и постоянство скорости света позволяет вывести из принципа Ферма все три закона геометрической оптики.
Закон прямолинейного распространения
Поскольку свет распространяется в однородной среде с постоянной скоростью, то минимальность времени становится эквивалентной минимальному расстоянию. Поэтому доказательство
закона прямолинейного распространения света
из принципа Ферма тривиально: Свет в однородной среде движется по кратчайшему расстоянию, соединяющему две точки, т.е. по отрезку прямой.
Закон отражения
Для доказательства закона отражения света можно обратиться к рисунку. Если отразить точечный источник S в зеркале, то для любой точки будет верно равенство длин отрезков: SR» = S»R» . Поэтому время прохождения света по пути S → R» → А будет равно времени прохождения света по пути S» → R» → А . Согласно принципу Ферма свет будет распространяться по «кратчайшему расстоянию», а из всех подобных расстояний минимальное будет для пути
S» → R → А
, когда точка будет находиться на отрезке S»А , соединяющем мнимое изображение источника и точку наблюдения (глаз). Не трудно видеть, что для этой точки угол падения равен углу отражения.
Это доказательство, естественно, не является строгим. По старой доброй традиции вставлю фразу: «Пытливый читатель может провести строгое доказательство самостоятельно». Перечислю лишь теоремы планиметрии, которые в нем использовались:
  • Признак равенства прямоугольных треугольников;
  • Неравенство треугольника;
  • Теорема о равенстве вертикальных углов;
На самом деле, в законе отражения есть еще фраза, которую многие часто забывают: падающий и отраженный лучи должны лежать в одной плоскости . Таким образом полное доказательство закона отражения не возможно без привлечения стереометрии. Эту часть доказательства «пытливый читатель также может провести строгое доказательство самостоятельно»
Закон преломления
Доказательство закона преломления света исходя из принципа Ферма несколько более сложное, чем представленные выше.

Принцип Ферма представляет собой предельный случай принципа Гюйгенса-Френеля в волновой оптике для случая исчезающей малой длины волны света.

id=»tabs-1″>

В модели можно изменять следующие величины:

  • Показатели преломления двух сред;
  • Положение источника света;
  • Положение приемника света;
  • Положение точки на границе раздела двух сред.
При этом в модели автоматически вычисляются следующие величины:
  • скорости распространения света в обоих средах;
  • время прохождения света в каждой из сред
Скачать задание к работе
Управление интерактивной моделью
  • Изменить масштаб: «CTRL + колесо мыши» или «CTRL + «+»»–«CTRL + «–»»
  • Изменить позицию: перетащить при зажатой «CTRL + левая кнопка мыши»
  • Стереть все «следы»: «CTRL + F »

Скачать модель

Авторами моделей, отмеченных знаком © CC-BY-SA, Являются указанные на сайте лица. Интерактивные модели распространяются по лицензии Creative Commons Attribution-Share Alike 3.0
Attribution-ShareAlike (by-sa) — Лицензия «С указанием авторства — Копилефт». Эта лицензия позволяет другим перерабатывать, исправлять и развивать произведение даже в коммерческих целях при условии указания авторства и лицензирования производных работ на аналогичных условиях. Эта лицензия является копилефт-лицензией. Все новые произведения основанные на лицензированном под нею будут иметь аналогичную лицензию, поэтому все производные будет разрешено изменять и использовать в коммерческих целях. При воспроизведении работ, распространяемых по данной лицензии ссылка на сайт обязательна!

Возьмите литровую банку и монету. Положите монету под дно пустой банки. Она видна как сверху, так и через боковую стенку банки. Теперь налейте в банку воду. Монета видна сверху, но не видна через боковую стенку банки. Почему? Положите монету внутрь банки с водой. Что изменилось и почему?

Пьер Ферма сформулировал принцип (то есть, общее утверждение), которому подчиняется распространение света в различных средах. Принцип, как и аксиома не доказывается. Из него получаются следствия, которые проверяются опытным путем. Сформулируем его.

Пусть свет распространяется между двумя точками по некоторому пути. На элементе пути ΔS скорость света равнялась v. Она может быть различна на разных участках. Тогда затраченное на этот участок время Полное время распространения света равно сумме времен, потраченных на все участки. На математическом языке это записывается как Ферма предположил, что

это время должно быть минимальным из возможных . То есть, перебрав все траектории, соединяющие начальную и конечную точку, мы должны найти ту, время движения света по которой минимально. Именно по этому пути «пойдет» световой луч. Величина называется оптической длиной пути . Величину n нельзя вынести за знак суммы, потому что она может быть различной на разных участках пути. Именно оптическая длина пути, а не геометрическая длина, должна быть наименьшей. Отсюда же следует принцип изохронизма световых лучей. Если из точки А в точку В свет распространяется по нескольким путям, то время распространения по ним одинаково.

Попробуем получить из этого принципа аксиомы геометрической оптики.

Прямолинейное распространение луча в однородной среде. Если луч движется из А в В без отражений в среде с постоянным показателем преломления n, то Это означает, что нужно выбрать путь из А в В минимальной геометрической длины . Ясно, что это будет прямая линия.

Закон зеркального отражения. Пусть свет пришел из А в В, испытав отражение в плоском зеркале.

Найдем точку О на зеркале, в которой произошло отражение. Отразив в зеркале точку В и получив точку В’, приходим к выводу, что длина ломаной AOB’ равна длине AOB. Очевидно, что AOB’ минимально по длине, когда это прямой отрезок. Получаем два вертикальных угла, один из которых обозначен двумя дугами, поэтому углы падения и преломления, обозначенные одной дугой, также должны быть равны. Точка О должна лежать в той вертикальной плоскости, в которой лежат перпендикуляры, опущенные из А и В на отражающую плоскость (иначе путь АОВ удлинится). Поэтому лучи АО и ОВ лежат в одной плоскости с перпендикуляром, опущенным в точку О.

Преломление луча на плоской границе. Пусть точки А и В лежат в средах, с показателями преломления n 2 и n 1 (n 2 >n 1), разделенные плоской границей. Легко сообразить, что в этом случае прямая АВ уже не будет соответствовать наименьшему времени. Поскольку, если мы сдвинем точку, в которой свет переходит из первой среды во вторую слегка налево, то путь, который свет пройдет в «медленной среде» сократится. А путь, пройденный в «быстрой» (имеющей меньший n) примерно на столько же удлинится. Результирующее время уменьшится. И так мы будем двигаться налево до тех пор, пока укорачивание времени в верхней среде не будет полностью компенсироваться удлинением его в нижней.

Второй рисунок показывает эту ситуацию. Если мы переместимся влево на малое расстояние вдоль границы A 1 A 2 , то геометрический путь в верхней среде сократится на A 2 B 2 , а оптический на n 2 A 2 B 2 , в нижней среде геометрический путь удлинится на A 1 B 1 , а оптический на n 1 A 1 B 1 . Мы достигнем минимума времени, если оптическую длину пути уже нельзя будет уменьшать такими шажками. То есть, укорачивание верхнего оптического пути равно удлинению нижнего n 1 A 1 B 1 =n 2 A 2 B 2 . По чертежу мы видим, что и где углы обозначены одной и двумя дугами соответственно. Из равенства получим

выполнение принципа Ферма приводит к известному закону преломления светового луча на границе разных сред.

Принцип Ферма представляет собой пример используемых в теоретической физике вариационных принципов. Для каждой траектории вычисляется определенная величина (в нашем случае – оптическая длина пути), после чего ищется такая траектория, на которой эта величина принимает минимальное (или максимальное) значение. Именно эта траектория и будет истинной. Подобно законам сохранения, вариационные принципы накладывают ограничения на происходящие события, делая их течение определенным. Почему законы сохранения и вариационные принципы «работают» — вопрос того же сорта, что и «Почему все тела притягиваются друг к другу всемирным тяготением».

Краткие теоретические сведения

Цель работы

Литература

Вопросы и тесты для самоконтроля и сдачи отчета

Вопросы для самоконтроля и сдачи отчета

Какую роль играет фотофильтры при работе фотоэлемента.

Вопросы допуска

4.5.1.Принцип работывакуумного фотоэлемента.

4.5.2. Как получить вольт-амперную характеристику вакуумного фотоэлемента.

4.5.4. Какие законы фотоэффекта мы сможем проверить на этой установке.

4.6.5. Фотоэлементы и их применение.

4.1.1.Определение законов фотоэффекта, длины волны.

а. – I тока насыщения пропорцианально световому потоку;

Е фото е — возрастает с частотой света

Если n 0 света

б. — I тока насыщения прямо пропорцианально мощности светового излучения.

Е фото е — линейно возрастает с частотой света и не зависит от мощности света

Если n 0 света

в. — I тока насыщения пропорцианально световому потоку;

Скорость фото е — зависит от мощности света.

N 0 света

4.7.2.Что мы называем током насыщения?

а. – при увеличении напряжения на аноде, ток возрастает.

б. – при постоянном напряжении на аноде увеличения фото е — не наблюдается;

в. – все е — , испущенные катодом, попадают на анод.

4.7.3.Уравнение Эйнштейна для явления фотоэффекта. Что такое фототок?

а. hn = ½ mnJ 0 2 – A; направление движения е — называется фототоком.

б. hn = ½ mnJ 0 2 + A; ток, возникающий под воздействием света

в. hw = ½ mJ 0 2 + A;

4.7.4. Что мы называем фотоэлектрическим эффектом?

а. испускание е — веществом при освещении.

б. испускание е — их сбор вокруг катода при воздействии света

в. испускание е — веществом под воздействием света.

4.8.1. Савельев И.В.Курс общей физики, т.4, 2004 г.

4.8.2. Кортнев А.В. и др. Практикум по физике, М., 1965 г.

5 . Определение показателей преломления твердых тел и жидкостей

Ознакомить с методами измерения показателей преломления жидких и твердых веществ.

Исследовать концентрационную зависимость показателей преломления жидких растворов и освоить метод определения неизвестной концентрации раствора с помощью рефрактометра.

Сделать численную и графическую проверку применимости формулы Лоренца к водному раствору глицерина.

Вычислить поляризуемость и эффективные радиусы молекул воды и глицерина.

Поведение света на границе раздела двух оптически различных сред определяется законом преломления, согласно которому падающий, преломленный луч и перпендикуляр, восстановленный в точке падания к поверхности раздела сред, лежат в одной плоскости. Отношение синуса угла падения a к синусу угла преломления, b есть постоянная для данных веществ величина, равная показателю преломления n 21 второй среды относительно первой. Закон преломления, как известно, был установлен экспериментально, а правильное теоретическое объяснение было дано Гюйгенсом на основании предложенного им принципа (принцип Гюйгенса).

Однако закон преломления теоретически можно получить с помощью более общего принципа, объясняющего ход световых лучей в разных ситуациях. Принцип Ферма, или принцип наименьшего времени утверждает, что из всех мыслимых траекторий между двумя точками действительной является та, которую свет проходит за минимальное время.

Пусть свет из точки А первой среды после преломления за границу раздела сред (плоскости Q) попадает в точку В второй среды.

Из принципа Ферма следует, что действительная траектория луча АОВ лежит в плоскости падения луча – плоскости, проведенной через падающий луч и нормаль к границе раздела в точке падения луча, и лишь найти положение точки О на линии раздела СД. Из рисунка 5.1 видно, что любую траекторию, не лежащую в точке падения Р, например, АМВ, свет пройдет за большее время. Действительно, отпустив перпендикуляр из точки М на плоскость Р, убеждаемся, что АМ > АО и MB > ОВ, так как гипотенуза всегда больше катета, и, следовательно, траекторию АМВ свет пройдет за большее время. Найдем положение точки О на плоскости падения. Обозначим СО через X. Траекторию АОВ свет пройдет за время t:

Рис. 5.1. К выводу закона преломления из принципа Ферми

где и — скорости света в первой и второй средах.

Чтобы в соответствии с принципом Ферма время t при некотором x было минимально, необходимо, чтобы первая производная в этой точке равнялась нулю.

Принцип Ферма – одна из наиболее важных теорем геометрической оптики. Несмотря на то, что он не используется непосредственно при расчете оптической системы (как, например, ), этот принцип используется для получения результатов, которые будет невозможно или очень сложно получить другим образом.

Этот принцип можно сформулировать следующим образом.

На рисунке 1.4 показан физически возможный путь лучей от точки до точки . Пусть длины отрезков вдоль луча будут равны .

Рисунок 1.4 – Оптическая длина пути.

Определим оптическую длину пути в любой среде как произведение пройденного лучом расстояния и коэффициента преломления:

(1.7)

где квадратные скобки использованы для того, чтобы различить оптическую длину пути от геометрического расстояния.

Принцип Ферма гласит, что оптическая длина пути, вдоль физически возможного пути луча – величина постоянная. Например, в простом случае плоской преломляющей поверхности (рисунок 1.5).

Рисунок 1 .5. Пример принципа Ферма.

У нас здесь есть луч, проходящий через две точки и . Предполагается, что преломляющая поверхность пересекается лучом в точке . По принципу Ферма, если мы запишем выражение для оптической длины пути как функцию от , а затем продифференцируем по относительно , то точка где дифференциал будет равен нулю совпадет с точкой . Это значит, что луч выбрал для своего пути кротчайшее расстояние.


Оптика — раздел физики, который занимается изучением природы света, законов распространения и взаимодействия с веществом.

Свет — это электромагнитное излучение в диапазоне длин волн от до (ф 0,4-0,79 мкм кр).

Видимый свет – это излучение в интервале длин волн: . Геометрическая оптика – раздел физики занимающийся изучением законов распространения света и получением изображений в оптических приборах. В основу геометрической оптики положено понятие светового луча (это линия указывающая направление распространения света) и световой пучок (это область пространства, в пределах которой распространяется свет). Световые пучки являются независимыми: каждый световой пучок при взаимном пересечении ведет себя самостоятельно, независимо от других пучков и не оказывает никакого влияния на другие пучки света. В основу г. о. положен принцип Ферма.

Принцип Ферма (первая формулировка): свет распространяется по такому пути, для прохождения которого ему требуется минимальное время. Пусть свет распространяется из точки 1 в точку 2 .Для прохождения элементарного участка dS свету потребуется время. Абсолютный показатель преломления среды , где с – скорость света, – скорость света в среде, то . Вторая формулировка: величина называется оптической длиной пути.Если среда однородна (n onst ), то L=nS , т. е. оптическая длина пути равнапроизведению показателя преломления среды на геометрическое расстояние между точками. Если заменить , т. е. пр. Ферма: свет распространяется по такому пути, длина которого минимальна, где s- геометрическая длина пути.

Оптические свойства вещества характеризуются величиной, называемой абсолютным показателем преломления n.

Абсолютный показатель преломления показывает во сколько раз скорость света в вакууме с больше скорости света в веществе v
Относительный показатель преломления равен отношению абсолютных показателей преломления в двух средах:

n 21 = n 2 /n 1 ; n 21 = v 1 /v 2 .


где v 1 и v 2 — скорость света в первой и во второй среде соответственно.

2. Основные законы геометрической оптики.

1) З-н прямолинейного распространения света: в однородной прозрачной среде свет распространяется прямолинейно.

2) З-н обратимости хода светового луча.(закон независимости световых лучей;)

3) З-н отражения света:

а)луч падающий, луч отраженный и перпендикуляр восстановленный в точку падения луча на границе раздела 2 сред, лежат в одной пл-ти.

б)угол падения= углу отражения.

4) закон независимости световых пучков. · (эффект, производимый отдельным пучком, не зависит от того , действуют ли одновременно остальные пучки или они устранены.

Разбивая световой поток на отдельные световые пучки (например, с помощью диафрагм), можно показать, что действие выделенных световых пучков независимо.)

5) З-н преломления света:

а)луч падающий, луч преломляющий и перпендикуляр восстановленный в точку падения луча на границе раздела 2 сред, лежат в одной плоскости.

б)отношение sin угла падения к sin угла преломления есть величина постоянная, равная относительному показателю двух сред, где – относительный показатель преломления, – абсолютный показатель света.

Закон отражения (рис. 7.3):

· отраженный луч лежит в одной плоскости с падающим лучом и перпендикуляром , проведенным к границе раздела двух сред в точке падения ;

· угол падения α равен углу отражения γ: α = γ

Для вывода закона отражения воспользуемся принципом Гюйгенса. Предположим, что плоская волна (фронт волны АВ с , падает на границу раздела двух сред (рис. 7.4). Когда фронт волны АВ достигнет отражающей поверхности в точке А , эта точка начнет излучать вторичную волну .

· Для прохождения волной расстояния ВС требуется время Δt = BC/υ. За это же время фронт вторичной волны достигнет точек полусферы, радиус AD которой равен: υ Δt = ВС. Положение фронта отраженной волны в этот момент времени в соответствии с принципом Гюйгенса задается плоскостью DC, а направление распространения этой волны – лучом II. Из равенства треугольников ABC и ADC вытекает закон отражения : угол падения α равен углу отражения γ.

Закон преломления (закон Снелиуса ) (рис. 7.5):

· луч падающий, луч преломленный и перпендикуляр, проведенный к границе раздела в точке падения, лежат в одной плоскости;

· отношение синуса угла падения к синусу угла преломления есть величина постоянная для данных сред.

Вывод закона преломления. Предположим, что плоская волна (фронт волны АВ ), распространяющаяся в вакууме вдоль направления I со скоростью с , падает на границу раздела со средой, в которой скорость ее распространения равна u (рис. 7.6).

Пусть время, затрачиваемое волной для прохождения пути ВС , равно Dt . Тогда ВС = с Dt. За это же время фронт волны, возбуждаемой точкой А в среде со скоростью u, достигнет точек полусферы, радиус которой AD = u Dt. Положение фронта преломленной волны в этот момент времени в соответствии с принципом Гюйгенса задается плоскостью DC, а направление ее распространения – лучом III. Из рис. 7.6 видно, что

Отсюда следует закон Снелиуса :

3. Применение принципа Ферма к доказательству законов отражения и преломления.

Принцип Ферма – основной принцип геометрической оптики . Простейшая форма принципа Ферма – утверждение, что луч света всегда распространяется в пространстве между двумя точками по тому пути, по которому время его прохождения меньше, чем по любому из всех других путей, соединяющих эти точки. Время прохождения светом расстояния l, заполненного средой с показателем преломления n , пропорционально оптической длине пути S ; S = l n для однородной среды, а при переменном n

S = ∫ndl,

Поэтому можно сказать, что принцип Ферма есть принцип наименьшей оптической длины пути . В первоначальной формулировке самого П. Ферма (около 1660) принцип имел смысл наиболее общего закона распространения света , из которого следовали все (к тому времени уже известные) законы геометрической оптики : для однородной среды он приводит к закону прямолинейности светового луча (в соответствии с геометрическим положением о том, что прямая есть кратчайшее расстояние между двумя точками), а для случая падения луча на границу различных сред из принципа Ферма можно получить законы отражения света и преломления света . В более строгой формулировке принцип Ферма представляет собой вариационный принцип , утверждающий, что реальный луч света распространяется от одной точки к другой по линии, по которой время его прохождения экстремально или одинаково по сравнению с временами прохождения по всем другим линиям, соединяющим эти точки. Это означает, что оптическая длина пути луча может быть не только минимальной, но и максимальной либо равной всем остальным возможным путям, соединяющим указанные точки. Примерами минимального пути служат упомянутые распространение света в однородной среде и прохождение светом границы двух сред с разными показателями преломления n . Все три случая (минимальности, максимальности и стационарности пути) можно проиллюстрировать, анализируя отражение луча света от вогнутого зеркала (рис.1).

Действительный путь света соответствует экстремальному времени распространения


Если зеркало имеет форму эллипсоида вращения, а свет распространяется от одного его фокуса Р к другому Q (причём путь без отражения невозможен), то оптическая длина пути луча PO» + O»Q по свойствам эллипсоида равна всем остальным возможным, например PO»» + О»» Q ; если на пути между теми же точками свет отражается от зеркала меньшей, чем у эллипсоида, кривизны (MM ), реализуется минимальный путь, если же большей (зеркало NN ) – максимальный. Условие экстремальности оптической длины пути сводится к требованию, чтобы была равна нулю вариация от интеграла

где А и В – точки, между которыми распространяется свет. Это выражение и представляет собой математическую формулировку принципа Ферма.

В волновой теории света принцип Ферма представляет собой предельный случай принципа Гюйгенса – Френеля и применим, когда можно пренебречь дифракцией света (когда длина световой волны достаточно мала по сравнению с характерными для задачи размерами): рассматривая лучи как нормали к волновым поверхностям, легко показать, что при всяком распространении света оптической длины их путей будут иметь экстремальные значения. Во всех случаях, когда необходимо учитывать дифракцию , принцип Ферма перестаёт быть применимым.

4.Преломоение света на плоской границе раздела 2-х сред. Полное внутреннее отражение

Если световой пучок падает на поверхность, разделяющую две прозрачные среды разной оптической плотности, например воздух и воду, то часть света отражается от этой поверхности, а другая часть — проникает во вторую среду. При переходе из одной среды в другую луч света изменяет направление на границе этих сред. Это явление называется преломлением света.

Законы преломления света.

Из всего сказанного заключаем:


1 . На границе раздела двух сред различной оптической плотности луч света при переходе из одной среды в другую меняет своё направление.
2. При переходе луча света в среду с большей оптической плотностью угол преломления меньше угла падения; при переходе луча света из оптически более плотной среды в среду менее плотную угол преломления больше угла падения.
Преломление света сопровождается отражением, причём с увеличением угла падения яркость отражённого пучка возрастает, а преломлённого ослабевает. Это можно увидеть проводя опыт, изображённом на рисунке. Следовательно, отражённый пучок уносит с собой тем больше световой энергии, чем больше угол падения.

Пусть MN -граница раздела двух про зрачных сред, например, воздуха и воды, АО -падающий луч, ОВ — преломленный луч, -угол падения, -угол преломления, -скорость распространения света в первой среде, — скорость распространения света во второй среде.

Первый закон преломления звучит так: отношение синуса угла падения к синусу угла преломления является постоянной величиной для данных двух сред:

, где — относительный показатель преломления (показатель преломления второй среды относительно первой).

Второй закон преломления света очень напоминает второй закон отражения света:

падающий луч, луч преломленный и перпендикуляр, проведенный в точку падения луча, лежит в одной плоскости.

Полное внутреннее отражение

Наблюдается для электромагнитных или звуковых волн на границе раздела двух сред, когда волна падает из среды с меньшей скоростью распространения (в случае световых лучей это соответствует бо́льшему показателю преломления).

С увеличением угла падения , угол преломления также возрастает, при этом интенсивность отражённого луча растет, а преломленного — падает (их сумма равна интенсивности падающего луча). При некотором критическом значении интенсивность преломленного луча становится равной нулю и происходит полное отражение света. Значение критического угла падения можно найти, положив в законе преломления угол преломления равным 90°:

5. Призмы

Призма — оптический элемент из прозрачного материала (например, оптического стекла) в форме геометрического тела — призмы, имеющий плоские полированные грани, через которые входит и выходит свет. Свет в призме преломляется. Важнейшей характеристикой призмы является показатель преломления материала, из которого она изготовлена. Виды призм: Дисперсионные призмы. Отражательные призмы. Поляризационные призмы.

Дисперсионные призмы Дисперсионные призмы используют в спектральных приборах для пространственного разделения излучений различных длин волн.

Отражательные призмы Отражательные призмы используют для изменения хода лучей, изменения направления оптической оси, изменения направления линии визирования, для уменьшения габаритных размеров приборов. Классифицируются отражательные призмы по нескольким признакам:


  • количеству отражений в призме

  • наличию или отсутствию «крыши»

  • характеру конструкции призмы

  • углу излома оптической оси
Также, особую нишу среди отражательных призм занимают составные призмы, — состоящие из нескольких частей, разделённых воздушными промежутками. Некоторые широко распространённые призмы получили собственные имена.

  • Призма Аббе

  • Призма Аббе-Порро

6. Тонкие линзы. Формула тонкой линзы

Линзой называется прозрачное тело, ограниченное двумя сферическими поверхностями. Если толщина самой линзы мала по сравнению с радиусами кривизны сферических поверхностей, то линзу называют тонкой . Линзы входят в состав практически всех оптических приборов. Линзы бывают собирающими и рассеивающими . Собирающая линза в середине толще, чем у краев, рассеивающая линза, наоборот, в средней части тоньше Линзы входят в состав практически всех оптических устройств . Линзы (Рис.3) делятся на собирающие и рассеивающие

Схема тонкой линзы

Рис.3,Собирающие (a) и рассеивающие (b) линзы и их условные обозначения.

Главной оптической осью линзы считается ось, прожодящая через центры кривизны её поверхностей. В тонкой линзе точки пересечения главной оптической оси с обеими поверхностями линзы сливаются в одну точку О.(Т.к. очень большие радиусы кривизны приближаются к плоскостям, то сферические поверхности теоретически сливаються в одну плоскость). Эта точка называется оптическим центром линзы. Тонкая линза имеет одну главную плоскость, которая общая для двух сферических поверхностей и проходит через центр призмы и перпендикулярна к главной оптической оси. Все прямые, проходящие через оптический центр линзы, называются побочными оптическими осями линзы. Важным является то, что все лучи, идущие через оптический центр линзы, не преломляются.

Поток монохроматических параллельных лучей или пучков лучей с осями их узких конусов, нормалльных к сферической границе раздела (к главной плоскости, называют парксиальными (приосевыми) пучками. При этом, пройдя через неё сходятся в главном фокусе линзы F 2 . Главные фокусы линзы лежат на главной оптической оси линзы. Точки, расположенные на главной оптической оси линзы с двух сторон оптического центра на равных расстояниях f 2 . , называются главными фокусами линзы . Плоскости, проходящие через главные фокусы f 2 линзы и перпендикулярные к её главной оптической оси , называются фокальными плоскостями линзы .

Формула тонкой линзы.

Формула тонкой линзы связывает между; собой три величины: расстояние от предмета до линзы d, расстояние от линзы до изображения f и фокус ное расстояние линзы F :

В формуле тонкой линзы фокусное расстояние ОF обозначается буквой F. Если линза собирающая, то > 0, если линза рассеивающая, то перед ставится знак «минус». Если изображение действительное, то > 0; если изображение воображаемое, то перед ставиться знак «минус». Все величины в формулу линзы подставляются в метрах.

7. Построение изображений в линзах

Опыт показывает, что параксиальные лучи света, выходящие из одной светящейся точки, после прохождения через линзу сходятся также в одной точке, которая является изображением светящейся точки. Поэтому для построения изображения точки достаточно взять два любых луча, но лучше те, ход которых после преломления заранее известен: 1 — луч, идущий через оптический центр; 2 — луч, параллельный главной оптической оси; 3 — луч, проходящий через передний фокус собирающей линзы (или продолжение луча 3 проходит через задний фокус рассеивающей линзы) (рис. 16.41).

Положение изображения действительного предмета и егоразмеры зависят от положения предмета относительно линзы. Пусть d — расстояние от предмета до линзы, f — расстояние от линзы до изображения. Построим изображение плоского предмета АВ , расположенного на различных расстояниях d от линзы. Если линза собирающая, то при d>2F (рис. 16.42) изображение действительное, перевернутое, уменьшенное,F

При F (рис. 16.43) изображение действительное, перевернутое, увеличенное, f>2F.


Рис. 16.43

При d (рис. 16.44) изображение мнимое, прямое, увеличенное, находится с той же стороны от линзы, что и сам предмет, но дальше предмета (f>d).


Рис. 16.44

В рассеивающей линзе (рис. 16.45) изображение действительного предмета всегда мнимое, прямое, уменьшенное, находится между линзой и ее фокусом со стороны изображаемого предмета. 

8.Глаз как оптический прибор. Лупа, Микроскоп, фотоаппарат.

Глаз. Основным источником зрения является глазное яблоко, за зрачком находится хрусталик, а сзади сетчатка. Оптическую роль в глазе выполняет элемент, имеющий форму двояковыпуклой линзы и наз-ся хрусталиком. К краям хрусталика прикреплены мышцы, которые сжимают или растягивают хрусталик, в результате меняются радиусы кривизны сферич. пов-ти хрусталика и соответственно фокусные расстояния. При изменении расстояния d до наблюдаемого объекта, расстояние f от хрусталика до сетчатки остается неизменным, а меняется фокусное расстояние. Недостатки зрения – близорукость и дальнозоркость.

Лупой называют собирающую тонкую линзу с малым фокусным расстоянием (5-10 см).увеличение лупы: , расстояние наилучшего зрения.

Принцип ферма законы отражения преломления. Доказательство закона преломления света с помощью принципа ферма. Явления преломления и отражения

Принцип Ферма

При́нцип Ферма́ (принцип наименьшего времени Ферма ) в геометрической оптике — постулат, предписывающий лучу света двигаться из начальной точки в конечную точку по пути, минимизирующему (реже — максимизирующему) время движения (или, что то же самое, минимизирующему оптическую длину пути). В более точной формулировке : свет выбирает один путь из множества близлежащих, требующих почти одинакового времени для прохождения; другими словами, любое малое изменение этого пути не приводит в первом порядке к изменению времени прохождения.

Этот принцип, сформулированный в I в. Героном Александрийским для отражения света, в общем виде был сформулирован Пьером Ферма в 1662 году в качестве самого общего закона геометрической оптики. В разнообразных конкретных случаях из него следовали уже известные законы: прямолинейность луча света в однородной среде, законы отражения и преломления света на границе двух прозрачных сред.

Принцип Ферма представляет собой предельный случай принципа Гюйгенса-Френеля в волновой оптике для случая исчезающей малой длины волны света.

Принцип Ферма является одним из экстремальных принципов в физике .

Примечания

Литература

  • Краткий словарь физических терминов / Сост. А. И. Болсун, рец. М. А. Ельяшевич. — Мн. : Вышэйшая школа, 1979. — С. 364-365. — 416 с. — 30 000 экз.

Ссылки

Wikimedia Foundation . 2010 .

  • Экстремальный принцип
  • Ифкуиль

Смотреть что такое «Принцип Ферма» в других словарях:

    принцип Ферма — — Тематики нефтегазовая промышленность EN Fermat s lawFermat s principle … Справочник технического переводчика

    принцип Ферма — Ferma principas statusas T sritis fizika atitikmenys: angl. Fermat’s law; Fermat’s principle vok. Fermatsches Prinzip, n rus. принцип Ферма, m pranc. principe de Fermat, m … Fizikos terminų žodynas

    Ферма принцип — Принцип Ферма на примере эллиптических поверхностях Объяснение закона Снелла при помощи принципа Ферма. Принцип Ферма (принцип наименьшего времени Ферма) в геометрической оптике постулат, предписывающий лучу света двигаться из начальной точки в… … Википедия

    Ферма Пьер — (Fermat) (1601 1665), французский математик, один из создателей аналитической геометрии и теории чисел (теоремы Ферма). Труды по теории вероятностей, исчислению бесконечно малых и оптике (принцип Ферма). * * * ФЕРМА Пьер ФЕРМА (Fermat) Пьер (1601 … Энциклопедический словарь

    ФЕРМА ПРИНЦИП — ФЕРМА ПРИНЦИП: действительный путь распространения света из одной точки в другую есть тот путь, для прохождения которого свету требуется минимальное (или максимальное) время по сравнению с любым др. геометрически возможным путем между теми же… … Энциклопедический словарь

    ФЕРМА — ФЕРМА (Fermat) Пьер де (1601 65), французский математик. Вместе с Блезом ПАСКАЛЕМ сформулировал теорию вероятности и, доказав, что свет перемещается по самой короткой оптической траектории (принцип Ферма), стал основателем геометрической оптики … Научно-технический энциклопедический словарь

    Ферма принцип{:} — действительный путь распространения света из одной точки в другую есть тот путь, для прохождения которого свету требуется минимальное (или максимальное) время по сравнению с любым другим геометрически возможным путём между теми же точками.… … Энциклопедический словарь

    ФЕРМА (Fermat) Пьер — (1601 65) французский математик, один из создателей аналитической геометрии и теории чисел (теоремы Ферма). Труды по теории вероятностей, исчислению бесконечно малых и оптике (принцип Ферма) … Большой Энциклопедический словарь

Доказательство
закона отражения света
из принципа Ферма

Принцип Ферма (принцип наименьшего времени Ферма) в геометрической оптике — постулат, предписывающий лучу света двигаться из начальной точки в конечную точку по пути, минимизирующему (реже — максимизирующему) время движения (или, что то же самое, минимизирующему оптическую длину пути).

Этот принцип, сформулированный в I в. Героном Александрийским для отражения света, в общем виде был сформулирован Пьером Ферма около 1660 года в качестве самого общего закона геометрической оптики. В разнообразных конкретных случаях из него следовали уже известные законы: прямолинейность луча света в однородной среде, законы отражения и преломления света на границе двух прозрачных сред .

Законы геометрической оптики и принцип Ферма

Конечность и постоянство скорости света позволяет вывести из принципа Ферма все три закона геометрической оптики.
Закон прямолинейного распространения
Поскольку свет распространяется в однородной среде с постоянной скоростью, то минимальность времени становится эквивалентной минимальному расстоянию. Поэтому доказательство закона прямолинейного распространения света из принципа Ферма тривиально: Свет в однородной среде движется по кратчайшему расстоянию, соединяющему две точки, т.е. по отрезку прямой.
Закон отражения
Для доказательства закона отражения света можно обратиться к рисунку. Если отразить точечный источник S в зеркале, то для любой точки будет верно равенство длин отрезков: SR» = S»R» . Поэтому время прохождения света по пути S → R» → А будет равно времени прохождения света по пути S» → R» → А . Согласно принципу Ферма свет будет распространяться по «кратчайшему расстоянию», а из всех подобных расстояний минимальное будет для пути S» → R → А , когда точка будет находиться на отрезке S»А , соединяющем мнимое изображение источника и точку наблюдения (глаз). Не трудно видеть, что для этой точки угол падения равен углу отражения.
Это доказательство, естественно, не является строгим. По старой доброй традиции вставлю фразу: «Пытливый читатель может провести строгое доказательство самостоятельно». Перечислю лишь теоремы планиметрии, которые в нем использовались:
  • Признак равенства прямоугольных треугольников;
  • Неравенство треугольника;
  • Теорема о равенстве вертикальных углов;
На самом деле, в законе отражения есть еще фраза, которую многие часто забывают: падающий и отраженный лучи должны лежать в одной плоскости . Таким образом полное доказательство закона отражения не возможно без привлечения стереометрии. Эту часть доказательства «пытливый читатель также может провести строгое доказательство самостоятельно»
Закон преломления
Доказательство закона преломления света исходя из принципа Ферма несколько более сложное, чем представленные выше.

Принцип Ферма представляет собой предельный случай принципа Гюйгенса-Френеля в волновой оптике для случая исчезающей малой длины волны света.

id=»tabs-1″>

В модели можно изменять следующие величины:

  • Показатели преломления двух сред;
  • Положение источника света;
  • Положение приемника света;
  • Положение точки на границе раздела двух сред.
При этом в модели автоматически вычисляются следующие величины:
  • скорости распространения света в обоих средах;
  • время прохождения света в каждой из сред
Скачать задание к работе
Управление интерактивной моделью
  • Изменить масштаб: «CTRL + колесо мыши» или «CTRL + «+»»–«CTRL + «–»»
  • Изменить позицию: перетащить при зажатой «CTRL + левая кнопка мыши»
  • Стереть все «следы»: «CTRL + F »

Скачать модель

Авторами моделей, отмеченных знаком © CC-BY-SA, Являются указанные на сайте лица. Интерактивные модели распространяются по лицензии Creative Commons Attribution-Share Alike 3.0
Attribution-ShareAlike (by-sa) — Лицензия «С указанием авторства — Копилефт». Эта лицензия позволяет другим перерабатывать, исправлять и развивать произведение даже в коммерческих целях при условии указания авторства и лицензирования производных работ на аналогичных условиях. Эта лицензия является копилефт-лицензией. Все новые произведения основанные на лицензированном под нею будут иметь аналогичную лицензию, поэтому все производные будет разрешено изменять и использовать в коммерческих целях. При воспроизведении работ, распространяемых по данной лицензии ссылка на сайт обязательна!

В основу геометрической оптики может быть положен принцип, установленный французским математиком Ферма в середине 17 столетия. Из этого принципа вытекают законы прямолинейного распространения света, отражения и преломления света. В формулировке самого Ферма принцип гласит, что свет распространяется по такому пути, для прохождения которого ему требуется минимальное время.

Пусть луч распространяется из точки 1 в точку пространства 2 (рис.1.7). Разобьем траекторию распространения света на прямолинейные участки, на которых показатель преломления будет константой, тогда чтобы свету пройти путь
требуется время

,

Следовательно, время, затрачиваемое светом на прохождение пути 1-2 равно

Величина имеет размерность длины и эту величину называют оптическим ходом луча или оптической длиной пути света

(1,9)

В однородной изотропной среде оптическая длина пути света равна

(1.10)

Пропорциональность времени tпрохождения оптической длине пути лучаLдает возможность сформулировать принцип Ферма следующим образом: свет распространяется по такому пути, оптическая длина которого экстремальна. Из принципа Ферма вытекает обратимость хода световых лучей. Действительно, оптический путь, который минимален в случае распространения света из точки 1 в точку 2, окажется минимальным и в случае распространения света из точки 2 в точку 1.

С помощью принципа Ферма можно доказать законы геометрической оптики, например, закон преломления света.

Доказательство закона преломления света с помощью принципа Ферма

Траектория по которой луч света из точки А, нкаходящейся в среде с показателем преломления n 1 , попадает в точку В, расположенную в среде с показателем преломленияn 2, может быть разной, но нам нужно показать, что луч будет распространяться по такому пути, на который он затратит минимальное время.

Опустим из точек А и В перпендикуляры на границу раздела двух сред и расстояния от точек до границы раздела обозначим соответственно а 1 и а 2 .

Так как точка перехода луча из одной среды в другую зависит от того по какой траектории будет распространяться луч света, то расстояние от первого перпедикуляра до точки падения (см.рис 1.8) обозначим x. Расстояние между опущенными перпендикулярами обозначимb.

Рис.1.8

Оптический путь луча будет состоять из двух частей, так как он распространяется в двух разныз средах:

Так как время распространения света из точки А в точку Bдолжно быть минимально, то оптический путь должен быть экстремален, т.е. первая производная оптического пути по времени должна быть равна нулю:

(1.11)

, а

Поэтому из условия (1,11) получаем

(1.12)

Т.е. закон преломления света доказан.

Полное внутреннее отражение, световоды (эндоскопы) .

Из формулы (1.12) видно, что при переходе света из оптически более плотной среды в оптически менее плотную луч удаляется от нормали к поверхности раздела сред. Увеличение угла падения сопровождается более быстрым ростом угла преломления ¦
и при некотором ¦ значении угла, котором преломленный луч пойдет по границе раздела двух сред, т.е. угол
достигает значения равного
, В этом случае угол паденияназывается предельным углом падения и определяется

(1.13)

Энергия, которую несет с собой падающий луч, распределяется между отраженным и преломленным лучами. По мере увеличения угла падения интенсивность отраженного луча растет, интенсивность же преломленного луча убывает, обращаясь в нуль при предельном угле. При углах падения, заключенных в пределах от предельного угла падения
до
, световая волна проникает во вторую среду на расстояние порядка длины волныи затем возвращается в первую среду. Это явление называется полным внутренним отражением (см.рис.1.9).

Явление полного внутреннего отражения находит применение во многих оптических устройствах. Наиболее интересным и практически важным применением является создание волоконных световодов , которые представляют собой тонкие (от нескольких микрометров до миллиметров) произвольно изогнутые нити из оптически прозрачного материала (стекло, кварц). Свет, попадающий на торец световода, может распространяться по нему на большие расстояния за счет полного внутреннего отражения от боковых поверхностей. Проверьте на опыте будет ли свет от красной лампочки распространяться по изогнутой струе воды.

Явление полного внутреннего отражения лежит в основе волоконной оптики. Свет, попадая внутрь прозрачного волокна, окруженного веществом с меньшим показателем преломления, многократно отражается и распространяется вдоль этого волокна. Диаметр этих тонких стеклянных или пластиковых волокон может быть доведен до нескольких микрометров. Для передачи больших световых потоков и сохранения гибкости светопроводящей системы отдельные волокна собираются в пучки (жгуты) – световоды, свет по световоду может передаваться почти без потерь. Рис1.10 демонстрирует, как распространяется свет по тонкому волокну, испытывая только скользящие отражения от стенок, т.е. претерпевая полное внутреннее отражение.

Если световоду придать сложную форму, то угол падения обычно превышает предельный, и свет будет передан от одного торца световода до другого практически без ослабления. Этот эффект используется в декоративных светильниках и при подсветке струй в фонтане. Волоконная оптика широко используется в медицине. Например, для визуального исследования внутренних полых органов используются гибкие гастроскопы, эндоскопы. С помощью световодов осуществляется передача лазерного излучения во внутренние ткани и органы с целью лечебного воздействия. На рис. 1.12 показаны различные способы подведения лазерного излучения к ткани: 1 — лазерный луч нацелен на ткань через систему диафрагм и линз; 2 — луч подводится через систему подвижных зеркал; 3 — луч проводится по гибкому пустотелому световоду с внутренней зеркальной поверхностью;

4 — луч проводится через гибкий кварцевый световод и дистанционно нацелен на ткань.

Рис. 1.12. Способы подведения лазерного излучения к ткани

Примером природной волоконнооптической системы является сетчатка человеческого глаза. Попадая на сетчатку, свет воспринимается светочувствительными элементами (волокнами двух типов: палочками и колбочками). Этот слой подобен волоконнооптическому устройству. У травянистых растений стебель играет роль световода, подводящего свет в подземную часть растения. Клетки стебля образуют параллельные колонки, напоминая этим конструкцию промышленных световодов. Если освещать такую колонку,рассматривая ее через микроскоп, то видно, что ее стенки при этом остаются темными, а внутренность каждой клетки ярко освещена. Глубина, на которую доставляется таким способом свет, не превышает 4-5 см. Но и такого короткого световода достаточно, чтобы обеспечить светом подземную часть травянистого растения.

Заключение

    Итак, свет обладает свойствами электромагнитной волны и потока фотонов, свойства неразделимы и в одних явлениях преобладает одно свойство, а в других другое, что связано с длиной световой волны.

    В анизотропной среде абсолютный показатель преломления зависит от направления распространения световой волны.

    В законах геометрической оптике используются чисто математические представления о лучах, не рассматривается природа света, законы работают при .

    Принцип Ферма является наиболее общим законом геометрической оптике, из этого закона могут быть выведены законы отражения и преломления света. Принцип Ферма определяет оптический путь луча и обратимость хода лучей.

    Закон полного внутреннего отражения позволяет понять принципы работы световодов (эндоскопов)

Ст. преподаватель кафедры___________________________

(наименование кафедры)

_______________________ ________________________

(ученая степень, ученое звание, подпись) (И.О.Ф.)

До сих пор мы рассматривали все соотношения геометрической оптики и законы действия оптических инструментов как следствие основных законов отражения и преломления света, сформулированных в § 8.

Однако, как показал около 300 лет назад Ферма, эти основные законы могут быть в свою очередь выведены из одного более общего принципа. Принцип Ферма утверждает, что свет распространяется по такому пути, при котором время, необходимое для прохождения света от одной точки до другой, имеет наименьшее или наибольшее значение (экстремум) 1).

Ферма считал свой принцип проявлением еще более общего принципа целесообразности: «природа всегда придерживается кратчайшего пути». Такое телеологическое толкование принципа Ферма было широко распространено в XVII и XVIII вв. и использовалось для доказательства существования бога. Однако, кроме всего прочего, такой трактовке резко противоречат все случаи, соответствующие наибольшему времени. Ниже показано, что принцип Ферма является просто одним из следствий волновой природы света, но справедлив только в области применимости методов геометрической оптики.

Если тело имеет показатель преломления то скорость света этом теле равна где с — скорость света в пустоте. Поэтому

время, в течение которого свет проходит расстояние I в среде с показателем преломления определяется соотношением

Произведение геометрического пути I на показатель преломления называют оптическим путем. Пусть свет проходит несколько сред с показателями преломления (рис. 39). Из точки А свет попадает в В по такому пути для которого время

имеет наибольшее или наименьшее значение.

Так как скорость света с в пустоте есть величина постоянная, то можно сформулировать принцип Ферма так: между точками оптический путь имеет минимальное или максимальное значение.

Из принципа Ферма легко вывести законы отражения и преломления.

Рис. 39. К принципу Ферма,

Рис. 40. Закон отражения как следствие принципа Ферма

В случае отражения оптический путь пропорционален геометрическому пути, так как распространение света происходит в одной среде. Поэтому применение принципа Ферма к отражению света сводится к решению чисто геометрических задач.

Пусть требуется найти кратчайший путь из точки при условии, что путь должен проходить через точку, лежащую на поверхности зеркала (рис. 40). Иначе говоря, требуется найти такую точку О на зеркале, чтобы путь был минимальным. Возьмем точку А, симметричную с А относительно поверхности зеркала Для любой точки отрезок следовательно, Отсюда становится очевидным, что искомой точкой будет точка О, лежащая на пересечении прямой В А

с поверхностью зеркала Из этого способа построения точки О получается закон отражения:

В то время как при отражении от плоского зеркала оптический путь имеет минимальное значение, в других случаях он может быть максимальным. Так, при отражении от вогнутых зеркал имеет место как наименьший оптический путь, так и наибольший. Пусть (рис. 41) — вогнутое зеркало, две точки, между которыми нужно найти оптический путь луча, отражающегося от зеркала

Рис. 41. При отражении от кривого зеркала оптический путь имеет иногда наименьшее, а иногда наибольшее значение.

Рис. 42. Вывод закона преломления из принципа Ферма.

Построим эллипсоид вращения с фокусами касательный к поверхности Из геометрического свойства эллипса следует, что все лучи, исходящие из А, собираются в точке В. Следовательно, искомый оптический путь проходит через точку касания Легко показать, что путь больше любого другого В самом деле, Последняя же сумма по геометрическому свойству эллипса равна Таким образом, рассмотренный случай дает пример наибольшего оптического пути. В случае вогнутого зеркала касательного к эллипсу снаружи, отражение происходит при минимальном оптическом пути, так же как в случае плоского зеркала и выпуклого

Когда свет проходит через границу (рис. 42) между двумя средами с показателями преломления то из условия минимума или максимума оптического пути

получается закон преломления:

где углы с нормалью к поверхности

Если величина имеет для светового луча экстремальное значение, то это значит, что величина при бесконечно малых изменениях формы луча не должна практически меняться. В данном случае наблюдается то же и в максимуме или минимуме обычной функции от какого-либо аргумента: бесконечно малые изменения аргумента не изменяют значения функции (производная равна нулю). Разница только в том, что величина зависит не от какого-либо аргумента, а от формы луча Проведем через точку С, бесконечно близкую к С, луч согласно сказанному длина оптического пути V для этого луча должна быть равна длине оптического пути

Иными словами,

Проведем из А и в окружности радиусами и тогда ввиду малости можно считать, что

или, пользуясь полученным выше соотношением:

Рассматривая треугольники и как прямоугольные, получим:

Подставив эти выражения и сократив на получим:

Если рассмотреть действие любой оптической системы, дающей изображения, То на первый взгляд кажется, что мы имеем дело с резким нарушением принципа Ферма. Свет от любой точки предмета до ее изображения распространяется по бесчисленному числу различных путей. Все лучи, вышедшие из светящейся точки, собираются в ее изображении, преломляясь и отражаясь различным образом.

Однако оказывается, что и здесь все находится в полном согласии с принципом Ферма. Частным случаем экстремума какой-либо величины является постоянство этой величины. Время прохождения света по всем лучам, образующим изображение точки, будет одно и то же. Задача становится неопределенной, мы не можем выбрать луча, соответствующего минимальному или максимальному времени, и сказать, что все остальные запрещены. Все лучи, идущие через оптическую систему, равновозможны с точки зрения принципа Ферма. Как было указано, если мы поместим источник в один фокус

эллиптического зеркала, то все лучи соберутся в другом фокусе. Из геометрических свойств эллипса прямо следует, что оптический путь от одного фокуса до другого по всем лучам будет один и тот же в согласии с принципом Ферма.

При прохождении линзы (положительной) центральный луч проходит меньший геометрический путь, чем краевые лучи, но, пользуясь формулой (5), можно показать, что оптические пути всех лучей будут равны. Этот результат вполне понятен, ибо чем короче световой луч (рис. 17), тем больший путь этот луч должен пройти в стекле, где свет распространяется медленнее, чем в воздухе. Вот это замедление света в стекле и компенсирует укорочение геометрического пути луча. Можно поступить наоборот и из постоянства оптических путей получить формулу (5).

Рис. 43. Астрономическая рефракция.

Все сказанное о равенстве оптических путей, конечно, справедливо только для случаев идеального изображения, когда светящейся точке соответствует точечное изображение. Если же система не сводит всех лучей, вышедших из точки, опять в одну точку, то, естественно, нарушается постоянство оптического пути по всем лучам. Чем хуже будет качество изображения, тем сильнее будут расходиться значения для различных оптических путей. В современной теории ошибок оптических инструментов, основанной Гамильтоном, мерилом качества оптической системы служит величина отклонений от постоянства оптических путей для различных лучей.

Особенно интересны применения принципа Ферма в оптике сред с непрерывно меняющимся показателем преломления.

В случае среды с непрерывно меняющимся показателем преломления условие максимума или минимума оптического пути может быть написано как условие максимума или минимума интеграла, выражающего сумму оптических путей

где элемент геометрического пути, показатель преломления — функция координат. Путь светового луча оказывается в этом случае криволинейным. Так, например, в земной атмосфере наблюдаются явления рефракции и миража, происходящие вследствие непрерывного изменения плотности, а следовательно, и показателя преломления воздуха. Так как плотность воздуха с высотой убывает, то рефракция искривляет световой луч, идущий от звезды (рис. 43), делает его падающим на землю более

отвесно, и поэтому наблюдатель видит звезду в точке 5, находящейся выше истинного положения звезды

Миражи наблюдаются при возникновении резких изменений плотности воздуха с высотой, которые вызываются Температурными условиями. Над раскаленным песком пустыни воздух сильно нагрет и плотность его мала. Поэтому до известной высоты в таких условиях может наблюдаться рост плотности воздуха с высотой, т. е. рост показателя преломления воздуха с высотой. Такое необычное изменение показателя преломления вызывает искривление светового луча, изображенное на рис. 44, а. Световой луч своей выпуклостью обращен к Земле, а не от Земли. В этом случае наблюдателю кажется, что световые лучи исходят от перевернутого предмета или что световые лучи отражаются от плоского зеркала. Это плоское зеркало воспринимается как большая водная поверхность.

Рис. 44. Происхождение миража

Наоборот, при резком убывании плотности воздуха с высотой световые лучи искривляются, так же как и при рефракции, но в гораздо большей степени. В результате световые лучи, идущие от предметов, лежащих за горизонтом, огибают Землю и попадают в глаз наблюдателя (рис. 44, б). Наблюдатель видит далекие предметы, и они ему кажутся близкими.

Таким же «миражем», по сути дела, является прием сверхдальней радиопередачи, происходящий в результате полного внутреннего отражения коротких радиоволн от ионосферы (т. II, § 91).

Траектория по которой луч света из точки А, нкаходящейся в среде с показателем преломления n 1 , попадает в точку В, расположенную в среде с показателем преломления n 2, может быть разной, но нам нужно показать, что луч будет распространяться по такому пути, на который он затратит минимальное время.

Опустим из точек А и В перпендикуляры на границу раздела двух сред и расстояния от точек до границы раздела обозначим соответственно а 1 и а 2 .

Так как точка перехода луча из одной среды в другую зависит от того по какой траектории будет распространяться луч света, то расстояние от первого перпедикуляра до точки падения (см.рис 1.8) обозначим x. Расстояние между опущенными перпендикулярами обозначим b.

Оптический путь луча будет состоять из двух частей, так как он распространяется в двух разныз средах:

Так как время распространения света из точки А в точку B должно быть минимально, то оптический путь должен быть экстремален, т.е. первая производная оптического пути по времени должна быть равна нулю:

(1.11)

, а

Поэтому из условия (1,11) получаем

(1.12)

Т.е. закон преломления света доказан.

Полное внутреннее отражение, световоды (эндоскопы) .

Из формулы (1.12) видно, что при переходе света из оптически более плотной среды в оптически менее плотную луч удаляется от нормали к поверхности раздела сред. Увеличение угла падения сопровождается более быстрым ростом угла преломления ¦ и при некотором ¦ значении угла , котором преломленный луч пойдет по границе раздела двух сред, т.е. угол достигает значения равного , В этом случае угол падения называется предельным углом падения и определяется

(1.13)

Энергия, которую несет с собой падающий луч, распределяется между отраженным и преломленным лучами. По мере увеличения угла падения интенсивность отраженного луча растет, интенсивность же преломленного луча убывает, обращаясь в нуль при предельном угле. При углах падения, заключенных в пределах от предельного угла падения до , световая волна проникает во вторую среду на расстояние порядка длины волны l и затем возвращается в первую среду. Это явление называется полным внутренним отражением (см.рис.1.9).

Явление полного внутреннего отражения находит применение во многих оптических устройствах. Наиболее интересным и практически важным применением является создание волоконных световодов , которые представляют собой тонкие (от нескольких микрометров до миллиметров) произвольно изогнутые нити из оптически прозрачного материала (стекло, кварц). Свет, попадающий на торец световода, может распространяться по нему на большие расстояния за счет полного внутреннего отражения от боковых поверхностей. Проверьте на опыте будет ли свет от красной лампочки распространяться по изогнутой струе воды.

Явление полного внутреннего отражения лежит в основе волоконной оптики. Свет, попадая внутрь прозрачного волокна, окруженного веществом с меньшим показателем преломления, многократно отражается и распространяется вдоль этого волокна. Диаметр этих тонких стеклянных или пластиковых волокон может быть доведен до нескольких микрометров. Для передачи больших световых потоков и сохранения гибкости светопроводящей системы отдельные волокна собираются в пучки (жгуты) – световоды, свет по световоду может передаваться почти без потерь. Рис1.10 демонстрирует, как распространяется свет по тонкому волокну, испытывая только скользящие отражения от стенок, т.е. претерпевая полное внутреннее отражение.

Если световоду придать сложную форму, то угол падения обычно превышает предельный, и свет будет передан от одного торца световода до другого практически без ослабления. Этот эффект используется в декоративных светильниках и при подсветке струй в фонтане. Волоконная оптика широко используется в медицине. Например, для визуального исследования внутренних полых органов используются гибкие гастроскопы, эндоскопы. С помощью световодов осуществляется передача лазерного излучения во внутренние ткани и органы с целью лечебного воздействия. На рис. 1.12 показаны различные способы подведения лазерного излучения к ткани: 1 — лазерный луч нацелен на ткань через систему диафрагм и линз; 2 — луч подводится через систему подвижных зеркал; 3 — луч проводится по гибкому пустотелому световоду с внутренней зеркальной поверхностью;

4 — луч проводится через гибкий кварцевый световод и дистанционно нацелен на ткань.

Рис. 1.12. Способы подведения лазерного излучения к ткани

Примером природной волоконнооптической системы является сетчатка человеческого глаза. Попадая на сетчатку, свет воспринимается светочувствительными элементами (волокнами двух типов: палочками и колбочками). Этот слой подобен волоконнооптическому устройству. У травянистых растений стебель играет роль световода, подводящего свет в подземную часть растения. Клетки стебля образуют параллельные колонки, напоминая этим конструкцию промышленных световодов. Если освещать такую колонку,рассматривая ее через микроскоп, то видно, что ее стенки при этом остаются темными, а внутренность каждой клетки ярко освещена. Глубина, на которую доставляется таким способом свет, не превышает 4-5 см. Но и такого короткого световода достаточно, чтобы обеспечить светом подземную часть травянистого растения.

Заключение

1. Итак, свет обладает свойствами электромагнитной волны и потока фотонов, свойства неразделимы и в одних явлениях преобладает одно свойство, а в других другое, что связано с длиной световой волны.

2. В анизотропной среде абсолютный показатель преломления зависит от направления распространения световой волны.

3. В законах геометрической оптике используются чисто математические представления о лучах, не рассматривается природа света, законы работают при l®0.

4. Принцип Ферма является наиболее общим законом геометрической оптике, из этого закона могут быть выведены законы отражения и преломления света. Принцип Ферма определяет оптический путь луча и обратимость хода лучей.

5. Закон полного внутреннего отражения позволяет понять принципы работы световодов (эндоскопов)

Читайте также…

Закон — прямолинейное распространение — Большая Энциклопедия Нефти и Газа, статья, страница 2

Закон — прямолинейное распространение

Cтраница 2

Совместно с законом прямолинейного распространения света эти законы составляют основу геометрической оптики. Они устанавливают связь параметров отраженного света с оптич.  [16]

Одним из выражений закона прямолинейного распространения света является принцип Ферма, утверждающий, что свет всегда распространяется по наикратчайшим путям.  [17]

На каких наблюдениях основан закон прямолинейного распространения света.  [18]

Описанные выше явления нарушения закона прямолинейного распространения света получили название дифракции света.  [20]

Дифракцией называются отклонения от закона прямолинейного распространения света. Приближенный расчет интенсивности света после препятствий или отверстий в экране опирается на принцип Гюйгенса — Френеля, который дополняет геометрическое построение Гюйгенса ( см. разд. Согласно этому принципу, все точки произвольной поверхности, окружающей источники волн, являются источниками когерентных вторичных волн, а амплитуда колебаний волны в произвольной точке вне этой поверхности может быть получена как результат интерференции вторичных волн. Обычно в качестве такой поверхности выбирают фронт волны, тогда источники вторичных волн синфазны.  [21]

Описанные выше явления нарушения закона прямолинейного распространения света получили название дифракции света.  [23]

Известно, что согласно закону прямолинейного распространения свет в прозрачной однородной среде распространяется по прямым линиям, которые называются лучами. Но есть и отступления от закона прямолинейного распространения света, когда он подобно звуку огибает препятствия, встречающиеся на пути его распространений и проникает в область тени. Это явление носит название дифракции, В общем случае под дифракцией света понимают всякое отклонение от прямолинейного распространения света, когда это отклонение нельзя объяснить как результат отражения, преломления или изгибания световых лучей в средах с непрерывно меняющимся показателем преломления.  [24]

Образование полутени не противоречит закону прямолинейного распространения света, а, наоборот, подтверждает его. Ведь в данном случае источник света состоит из множества точек и каждая из них испускает лучи. В центральную область экрана не попадает свет ни от одной точки лампы, там наблюдается полная тень.  [25]

Основные законы геометрической оптики — закон прямолинейного распространения света в однородной среде, законы отражения и преломления света на границе двух сред — могут быть получены и с помощью принципа Ферма. Согласно этому принципу действительный путь луча света есть путь, для прохождения которого свету требуется экстремальное ( как правило, минимальное) время по сравнению с любым другим близким к действительному мыслимым путем между теми же точками. Хотя такая формулировка принципа Ферма и не вполне точна, она достаточна для понимания рассматриваемых ниже примеров.  [26]

Основные законы геометрической оптики — закон прямолинейного распространения света в однородной среде, законы отражения и преломления света на границе раздела двух сред — могут быть получены с помощью принципа Ферма. Согласно этому принципу действительный путь распространения монохроматического луча света есть путь, для прохождения которого свету требуется экстремальное ( как правило, минимальное) время по сравнению с любым другим близким к нему мыслимым путем между теми же точками.  [28]

Основные законы геометрической оптики — закон прямолинейного распространения света в однородной среде, законы отражения и преломления света на границе раздела двух сред — могут быть получены с помощью принципа Ферма: действительный путь распространения монохроматического луча света есть путь, для прохождения которого свету требуется экстремальное ( как правило, минимальное) время посравнениюслюбым другим мыслимым путем между теми же точками. Поскольку скорость света в среде с показателем преломления п равна.  [29]

Основные законы геометрической оптики — закон прямолинейного распространения света в однородной среде, законы отражения и преломления света на границе раздела двух сред — могут быть получены с помощью принципа Ферма. Согласно этому принципу действительный путь распространения монохроматического луча света есть путь, для прохождения которого свету требуется экстремальное ( как правило, минимальное) время по сравнению с любым другим мыслимым путем между теми же точками. Такая формулировка принципа Ферма на самом деле не вполне верна. Согласно принципу Ферма оптический путь должен сравниваться не с любым другим, а с ближайшим.  [30]

Страницы:      1    2    3    4

Геометрическая оптика — Закон

Геометрическая оптика

Геометрическая (лучевая) оптика использует идеализированное представление о световом луче – бесконечно тонком пучке света, распространяющемся прямолинейно в однородной изотропной среде, а также представления о точечном источнике излучения, равномерно светящем во все стороны. – длина световой волны, – характерный размер предмета, находящегося на пути волны. Геометрическая оптика является предельным случаем волновой оптики и ее принципы выполняются при соблюдении условия:

<<1,

т. е. геометрическая оптика, строго говоря, применима лишь к бесконечно коротким волнам.

Принцип Ферма

Основным принципом геометрической оптики является принцип наименьшего времени, которой был высказан французским физиком и математиком Пьером Ферма в 1662году. Также этот принцип называют принципом кратчайшего оптического пути: луч, распространяющийся между двумя точками, выбирает пути, требующий минимального времени.

Для однородной среды этот принцип приводит к закону прямолинейного распространения согласно геометрической аксиоме о том, что прямая есть кратчайшее расстояние между двумя точками.

Для прохождения участка пути свету требуется время , где – скорость света в заданной среде. – время, затраченное на элементе : .

оптическая длина пути (ОДП).

В однородной среде ОДП есть геометрическая длина пути, умноженная на : .

Из принципа Ферма вытекает обратимость световых лучей: поскольку оптический путь, который минимален в случае распространения света из т.1 в т.2, окажется минимальным и при распространении в обратном направлении. Следовательно, луч, пущенный навстречу лучу, проделавшему путь от т.1 к т.2, пройдет по тому же пути, но в обратном направлении.

В основе геометрической оптики лежит так же принцип независимости световых лучей: лучи при перемещении не возмущают друг друга. Поэтому перемещения лучей не мешают каждому из них распространяться независимо друг от друга.

Законы отражения и преломления света

Получим с помощью принципа Ферма законы отражения и преломления света. Пусть свет падает из точки А в точку В, отразившись от поверхности MN. Геометрическая длина произвольного пути (т. является зеркальным отражением т. A).

<.

Но .

Сделав дополнительные построения пунктирными линиями видим, что наименьшей длиной обладает путь луча, отразившегося в т.О.

– угол падения,

– угол отражения.

<

Но,

ЗАКОН ОТРАЖЕНИЯ: луч падающий, луч отраженный и перпендикуляр к поверхности раздела, восстановленный в точке падения луча, лежат в одной плоскости, называемой плоскостью падения; угол падения равен углу отражения .

Рассмотрим случай, когда точки А и В лежат в разных средах. Время прохождения луча от точки А до точки В:

Учитывая принцип Ферма, исследуем на экстремум величину :

.

Из рисунка видно, что полученную формулу можно преобразовать:

.

Отсюда:

, но (с – скорость света в вакууме, — скорость света в среде с показателем преломления n ( – абсолютный показатель преломления). С учетом этого:

, где – относительный показатель преломления второй среды относительно первой среды.

Закон преломления: луч падающий, луч преломленный и перпендикуляр к поверхности раздела, восстановленный из точки падения луча, лежат в одной плоскости – плоскости падения. Отношение синуса угла падения к синусу угла отражения равно отношению скоростей света в обеих средах.

Можно записать и так:

.

Из закона преломления, применяемого для случая >, следует:

, т.к. >, .

, возможно, что .

, т. е. угол преломления равен 90о, при этом преломленный луч скользит по границе раздела. В этом случае угол падения называют – предельным углом. При дальнейшем увеличение угла падения проникновение луча вглубь второй среды прекращается и наступает полное отражение.

Полное отражение находит

различные практические применения.

Поскольку для системы стекло-воздух

< 45о, то призмы позволяют поменять

ход луча.





Явление полного внутреннего

отражения используется в волоконной

оптике при создании световодов –

тонких прозрачных волокон.

Зеркала

Для понимания основных закономерностей в явлениях отражения наиболее целесообразно рассмотреть отражение в плоских и сферических зеркалах. Зеркала для оптических систем изготовляются в виде плоскостей или сфер из стекла, на поверхность которых наносится испарением в вакууме или химическим путем слой металла (серебра, алюминия, меди и др.), дающих высокий коэффициент отражения света. Вместе с этим применяется способ изготовления зеркал из цельного куска металла, например, алюминия. Изобразим на рисунке ход лучей при отражении света от плоской поверхности S:

Свет идет из точечного источника А. и – нормали к поверхности S. Из закона отражения света следует: , . Лучи и являются отраженными. Продолжим луч до пересечения с продолжением высоты h вниз. Из построения следует: . Значит треугольники ACD и равны между собой. Отсюда следует, что . Это же заключение можно было сделать из рассмотрения лучей и . Доказано: после отражения от плоского зеркала лучей, исходящих из точечного источника, они идут так, как будто вышли из мнимого источника, находящегося позади зеркала на перпендикуляре к его плоскости, равном расстоянию действительного источника от плоскости зеркала. В рассмотренном случае таким мнимым источником является точка .

Изображение произвольного предмета может быть определено путем построения изображения точек, из которых состоит предмет.

Плоские зеркала находят широкое применение в самых разнообразных оптических приборах. Важным применением плоских зеркал является поворот луча света точно в обратном направлении. Это достигается с помощью уголкового отражения, представляющего собой совокупность трех плоских зеркал, поставленных под прямым углом друг к другу.

Рассмотрим теперь явление отражения света от сферического зеркала. Вначале рассмотрим отражение от вогнутого зеркала S.

Введем существенное ограничение: будем рассматривать только прохождение лучей, незначительно удаленных от оси симметрии , которая называется оптической осью зеркала (аналогично оптической оси линзы, оптической системы). Также лучи называют параксиальными лучами, а совокупность явлений в оптических системах при таком ходе лучей получила название параксиальная оптика. Из-за малости угла наклона при данном рассмотрении можно значение тангенсов и синусов этих углов заменить значениями самих углов.

Точка является пересечением лучей и , отраженных от зеркала, и представляет собой изображение точки А. Введем обозначения:

О – вершина зеркала,

a = АО – расстояние от вершины зеркала до источника света,

b = – расстояние от вершины зеркала до изображения источника,

R = OC – радиус кривизны зеркала (совпадает с нормалью),

f = OF – фокусное расстояние,

h – расстояние т. М от оптической оси.

Будем считать отрезки вправо от точки О и вверх от оптической оси положительными, а влево и вниз – отрицательными. Из чертежа видно:

Преобразуем:

Откуда: .

Ввиду малости углов : , , .

После подстановки:

.

Если , то . Тогда F, в которой получается изображение в этом случае называется главным фокусом зеркала (или просто фокусом).

Расстояние f от т. F до вершины О зеркала S называется фокусным расстоянием, причем:

.

Таким образом, падающие на зеркало параллельные лучи после отражения собираются в его фокусе.

формула зеркала, f < 0, R < 0, фокус F действительный.

Рассмотрим теперь изображение в выпуклом зеркале.

Из Δ AMС:

;

Δ :

Отсюда:

Для углов: ,

, .

Подставляя, получим: . Формула для выпуклого зеркала такая же, как и для вогнутого. Если , то .

Точка F является мнимым фокусом выпуклого зеркала (f > 0, R > 0). После подстановки:

формула зеркала.

Линейным увеличением зеркала называют отношение линейных размеров изображения к линейным размерам предмета:

.

Тонкие линзы

Прежде, чем рассматривать линзы, рассмотрим преломление луча на сферической поверхности.

Здесь:

R – радиус кривизны,

a – расстояние от поверхности до предмета,

C – центр сферы,

b – расстояние от поверхности до изображения.

Согласно закону преломления:

Для параксиальных лучей: .

Кроме того: , /

Поэтому: .

Аналогично из /

Используя эти соотношения, получаем:

или .

Рассматривая случаи малых углов: , , , .

Полученное уравнение справедливо и для вогнутой поверхности, однако у a, b, R нужно поменять знак, поскольку изображение будет мнимым.

Используя полученные результаты, рассмотрим устройства, изготавливаемые из прозрачного вещества, ограниченные двумя сферическими поверхностями, вершины которых лежат на одной оси, называемой оптической осью. Такие устройства называют линзами. Если расстояние a от линзы до изображения много меньше расстояния l между поверхностями линз (толщиной линзы), то линзу называют тонкой. Для нее справедливо соотношение:

уравнение тонкой линзы.

– относительный показатель преломления.

Если , то лучи соберутся в фокусе (). Величина определяет положение второго или заднего фокуса линзы

.

Если , , – задний фокус расположен справа от линзы (двояковыпуклая линза).

Если , то получаем значение первого или переднего фокуса линзы.

– для двояковыпуклой – лежит слева от линзы.

С учетом сказанного:

формула тонкой линзы

Линзы бывают собирающими и рассеивающими. Первые из них обладают способностью собирать в точку лучи, исходящие из точечного источника, тогда как вторые такой способностью не обладают.

Линейным увеличением линзы называют отношение линейных размеров изображения к линейным размерам предмета:

.

Величина, обратная фокусному расстоянию, измеренному в метрах, называется оптической силой линзы измеряется в диоптриях:

;

-показатель преломления среды, в которой находится линза.

Если линза находится в воздухе:

.

В случае, если имеется система нескольких соприкасающихся тонких линз:

, где – заднее фокусное расстояние системы линз, – задние фокусные расстояния каждой из совокупности линз, составляющих систему.

Math.ru

Лев Васильевич Тарасов, Альдина Николаевна Тарасова

М.: Наука, 1982. 176 с.
Тираж 150000 экз.
Серия Библиотечка «Квант», выпуск 18

Загрузить (Mb)
djvu (5.76) pdf (-) ps (-) html (-) tex (-)

Книга состоит из девяти бесед, охватывающих широкий круг вопросов, связанных с преломлением света. Познакомившись с ней, читатаель убедится, как много удивительного, поучительного, полезного для практики может заключаться в, казалось бы, хорошо знакомом явлении преломления света. Оно позволило объяснить такие «загадки» природы, как миражи, радуги, гало, ложные солнца и другие; оно же помогло человеку «приручить» световой луч с тем, чтобы использовать его для практических целей. Популярно и увлекательно авторы рассказывают о призменных и линзовых оптических системах, о кристаллооптике, о волоконной оптике, позволяющей, образно говоря, произвольно изгибать световой луч, и о многом другом.

Содержание

Предисловие

Беседа первая. ЧТО ПРОИСХОДИТ СО СВЕТОВЫМИ ЛУЧАМИ НА ГРАНИЦЕ ДВУХ СРЕД?
    Кольцо на дне сосуда с водой
    Опыты Птолемея
    Установление закона преломления Снеллиусом
    Объяснение закона преломления Декартом; ошибка Декарта
    Принцип Гюйгенса
    Принцип Гюйгенса и закон преломления
    Принцип Ферма (принцип наименьшего времени)
    Вывод закона преломелния из принципа Ферма
    Применение принципа Ферма
    Полное внутреннее отражение света; предельный угол отражения
    Графический метод построения преломленных лучей
    Излучение Вавилова-Черенкова и законы преломелния и отражения

Беседа вторая. К КАКИМ ОБМАНАМ ЗРЕНИЯ ПРИВОДИТ РЕФРАКЦИЯ СВЕТА В ЗЕМНОЙ АТМОСФЕРЕ?
    Рефракция света в атмосфере; угол рефракции
    Ранние представления о рефракции света в атмосфере
    Рефракция света по Кеплеру
    Восстановление теории рефракции Ньютона по его переписке с Флемстидом
    Экспоненциальный закон убывания плотности атмосферы с высотой
    Своеобразие солнечных закатов; появление «слепой полосы»
    Мерцание звезд
    Искривление светового луча в оптически неоднородной среде
    Миражи

Беседа третья. КАК СВЕТОВОЙ ЛУЧ ПРОХОДИТ ЧЕРЕЗ ПРИЗМУ?
    Преломление светового луча в призме; угол отклонения луча
    Случаи симметричного и несимметричного хода луча в призме
    Рефрактометры
    Объясенение появления в оконном стекле двойных изображений отдаленных предметов
    Отражательные призмы
    Фотометр Люммера-Бродхуна
    Отражательная призма вместо отражающего зеркала лазерного резонатора
    Бипризма

Беседа четвертая. ПОЧЕМУ ПРИЗМА РАЗЛАГАЕТ СОЛНЕЧНЫЙ СВЕТ НА РАЗЛИЧНЫЕ ЦВЕТА?
    Дисперсия света
    Первые опыты с призмами; представления о причинах возникновения цветов до Ньютона
    Опыты Ньютона с призмами; ньютоновсая теория возникновения цветов
    Работы Эйлера; сопостовление цветам разных длин волн
    Открытие аномальной дисперсии света; опыты Кундта
    Замечания по поводу отражательной призмы
    Дисперсионные призмы; угловая дисперсия
    Спектральные приборы — монохроматоры и спектрометры; схема Фукса — Уодсворта
    Гёте против Ньютона

Беседа пятая. КАК ВОЗНИКАЕТ РАДУГА?
    Радуга глазами внимательного наблюдателя
    Развитие представлений о физике возникновения радуги — от Флетчера, Доминико и Декарта к Ньютону
    Объяснение возникновения радуги в ньютоновских «Лекциях по оптике»
    Ход светового луча в капле дождя
    Наибольший угол между направлениями падающих на каплю и выходящих из нее лучей
    Объяснение чередования цветов в основной и вторичной радугах
    Радуга на других планетах
    Причины возникновения гало; гало и радуга

Беседа шестая. КАК ПОЛУЧАЮТ ОПТИЧЕСКИЕ ИЗОБРАЖЕНИЯ?
    Получение изображение в камере-обскуре
    Получение изображения в линзовой системе
    Вывод формулы тонкой линзы на основе принципа Ферма
    Сферическая и хроматическая аберрации
    Действительные и мнимые изображения
    Собирающие и рассеивающие линзы
    Линза в оптически плотной среде
    Из ранней истории развития линзовых систем
    Изобретение зрительной трубы
    Ход лучей в трубе Галилея; угловое увеличение
    Астономические наблюдения Галилея
    «Диоптрика» Кеплера и последующие работы
    Ахроматическая линза Доллонда
    Зонная пластинка Френеля

Беседа седьмая. КАК УСТРОЕН ГЛАЗ?
    Две группы оптических приборов
    Строение и оптическая схема человеческого глаза
    Система, состоящая из лупы и глаза
    Развитие учения о зрении от Демокрита и Галена к Альхазену и Леонардо да Винчи
    Сопостовление глаза с камерой-обскурой в трудах Леонардо да Винчи
    Кеплер о роли хрусталика в глазе; объяснение Юнгом механизма аккомодации
    Дальнозоркость и близорукость
    Глаз как совершенное оптическое устройство
    Очки
    Использование линзовых систем для увеличения угла зрения
    Фасеточные глаза насекомых

Беседа восьмая. ПОЧЕМУ В КРИСТАЛЛАХ НАБЛЮДАЕТСЯ ДВОЙНОЕ ПРЕЛОМЛЕНИЕ СВЕТА?
    Открытие Бартолином двойного преломления света в кристалле исландского шпата
    Кристалл как оптически анизотропная среда
    Объяснение двойного лучепреломления в «Трактате о свете» Гюйгенса; обыкновенная и необыкновенная световые волны
    Построения Гюйгенса; скорость световой волны и лучевая скорость
    Опыты Гюйгенса с двумя кристаллами (на пороге открытия поляризации света)
    Объяснение Ньютоном результатов опытов Гюйгенса
    Исследование Малюса и Брюстера
    Поляризация света
    Дихроичные пластинки и поляризационные призмы
    Поворот плоскости поляризации света в полуволновой пластинке

Беседа девятая. ЧТО ТАКОЕ ВОЛОКОННАЯ ОПТИКА?
    Светящаяся водяная струя
    Световые лучи в прямом и изогнутом цилиндрических волокнах
    Лучи в коническом волокне
    Влияние изгиба волокна
    Градиентные оптические волокна
    Тонкие волокна
    Передача оптических изображений по волоконному жгуту
    Волоконный выравниватель светового поля
    Волоконный диссектор изображения в высокоскоростной фотографии
    Сетчатка глаза как волоконнооптическое устройство

ЗАКЛЮЧЕНИЕ
    Управление преломляющими свойствами вещества
    Электрооптический дефлектор
    Космические линзы


Загрузить (Mb)
djvu (5.76) pdf (-) ps (-) html (-) tex (-)


Основные понятия геометрической оптики. Принцип Ферма Оптика


  1. Основные понятия геометрической оптики. Принцип Ферма

Оптика— раздел физики, который занимается изучением природы света, законов распространения и взаимодействия с веществом.

Свет— это электромагнитное излучение в диапазоне длин волн от до (ф 0,4-0,79 мкм кр).

Видимый свет– это излучение в интервале длин волн: . Геометрическая оптика – раздел физики занимающийся изучением законов распространения света и получением изображений в оптических приборах. В основу геометрической оптики положено понятие светового луча (это линия указывающая направление распространения света) и световой пучок (это область пространства, в пределах которой распространяется свет). Световые пучки являются независимыми: каждый световой пучок при взаимном пересечении ведет себя самостоятельно, независимо от других пучков и не оказывает никакого влияния на другие пучки света. В основу г. о. положен принцип Ферма.

Принцип Ферма (первая формулировка): свет распространяется по такому пути, для прохождения которого ему требуется минимальное время. Пусть свет распространяется из точки 1 в точку 2 .Для прохождения элементарного участка dS свету потребуется время. Абсолютный показатель преломления среды , где с – скорость света, – скорость света в среде, то . Вторая формулировка:величина называется оптической длиной пути.Если среда однородна (nonst), то L=nS, т. е. оптическая длина пути равнапроизведению показателя преломления среды на геометрическое расстояние между точками. Если заменить , т. е. пр. Ферма: свет распространяется по такому пути, длина которого минимальна, где s- геометрическая длина пути.

Оптические свойства вещества характеризуются величиной, называемой абсолютным показателем преломления n. 

Абсолютный показатель преломления показывает во сколько раз скорость света в вакууме с больше скорости света в веществе v

n = c/v.


Относительный показатель преломления равен отношению абсолютных показателей преломления в двух средах:

n21 = n2/n1;    n21 = v1/v2.


где v1 и v2 — скорость света в первой и во второй среде соответственно.

2. Основные законы геометрической оптики.

1) З-н прямолинейного распространения света: в однородной прозрачной среде свет распространяется прямолинейно.

2) З-н обратимости хода светового луча.( закон независимости световых лучей;)

3) З-н отражения света:

а)луч падающий, луч отраженный и перпендикуляр восстановленный в точку падения луча на границе раздела 2 сред, лежат в одной пл-ти.

б)угол падения= углу отражения.

4) закон независимости световых пучков. ·     (эффект, производимый отдельным пучком, не зависит от того, действуют ли одновременно остальные пучки или они устранены.

      Разбивая световой поток на отдельные световые пучки (например, с помощью диафрагм), можно показать, что действие выделенных световых пучков независимо. )

5) З-н преломления света:

а)луч падающий, луч преломляющий и перпендикуляр восстановленный в точку падения луча на границе раздела 2 сред, лежат в одной плоскости.

б)отношение sin угла падения к sin угла преломления есть величина постоянная, равная относительному показателю двух сред, где – относительный показатель преломления, – абсолютный показатель света.

      Закон отражения (рис. 7.3):

       ·        отраженный луч лежит в одной плоскости с падающим лучом и перпендикуляром, проведенным к границе раздела двух сред в точке падения;

       ·        угол падения  α  равен углу отражения  γ:   α = γ

             Для вывода закона отражения воспользуемся принципом Гюйгенса. Предположим, что плоская волна (фронт волны АВ), распространяющаяся в вакууме вдоль направления I со скоростью с, падает на границу раздела двух сред (рис. 7.4). Когда фронт волны АВ достигнет отражающей поверхности в точке А, эта точка начнет излучать вторичную волну.

       ·       Для прохождения волной расстояния ВС требуется время             Δt = BC/υ. За это же время фронт вторичной волны достигнет точек полусферы, радиус AD которой равен:  υΔt = ВС. Положение фронта отраженной волны в этот момент времени в соответствии с принципом Гюйгенса задается плоскостью DC, а направление распространения этой волны – лучом II. Из равенства треугольников ABC и ADC вытекает закон отражения: угол падения  α  равен углу отражения  γ.

      Закон преломления (закон Снелиуса) (рис. 7.5):

       ·     луч падающий, луч преломленный и перпендикуляр, проведенный к границе раздела в точке падения, лежат в одной плоскости;

       ·     отношение синуса угла падения к синусу угла преломления есть величина постоянная для данных сред.

      Вывод закона преломления. Предположим, что плоская волна (фронт волны АВ), распространяющаяся в вакууме вдоль направления I со скоростью  с, падает на границу раздела со средой, в которой скорость ее распространения равна u  (рис. 7.6).

      Пусть время, затрачиваемое волной для прохождения пути ВС, равно Dt. Тогда ВС = сDt. За это же время фронт волны, возбуждаемой точкой А в среде со скоростью u, достигнет точек полусферы, радиус которой AD = uDt. Положение фронта преломленной волны в этот момент времени в соответствии с принципом Гюйгенса задается плоскостью DC, а направление ее распространения – лучом III. Из рис. 7.6 видно, что

       ,       т.е.       .

      Отсюда следует закон Снелиуса:

.

3. Применение принципа Ферма к доказательству законов отражения и преломления.

Принцип Ферма – основной принцип геометрической оптики. Простейшая форма принципа Ферма – утверждение, что луч света всегда распространяется в пространстве между двумя точками по тому пути, по которому время его прохождения меньше, чем по любому из всех других путей, соединяющих эти точки. Время прохождения светом расстояния l, заполненного средой с показателем преломления n, пропорционально оптической длине пути S; S = l•n для однородной среды, а при переменном n

S = ∫ndl,

Поэтому можно сказать, что принцип Ферма есть принцип наименьшей оптической длины пути. В первоначальной формулировке самого П. Ферма (около 1660) принцип имел смысл наиболее общего закона распространения света, из которого следовали все (к тому времени уже известные) законы геометрической оптики: для однородной среды он приводит к закону прямолинейности светового луча (в соответствии с геометрическим положением о том, что прямая есть кратчайшее расстояние между двумя точками), а для случая падения луча на границу различных сред из принципа Ферма можно получить законы отражения света и преломления света. В более строгой формулировке принцип Ферма представляет собой вариационный принцип, утверждающий, что реальный луч света распространяется от одной точки к другой по линии, по которой время его прохождения экстремально или одинаково по сравнению с временами прохождения по всем другим линиям, соединяющим эти точки. Это означает, что оптическая длина пути луча может быть не только минимальной, но и максимальной либо равной всем остальным возможным путям, соединяющим указанные точки. Примерами минимального пути служат упомянутые распространение света в однородной среде и прохождение светом границы двух сред с разными показателями преломления n. Все три случая (минимальности, максимальности и стационарности пути) можно проиллюстрировать, анализируя отражение луча света от вогнутого зеркала (рис.1).

Действительный путь света соответствует экстремальному времени распространения

Рис.1


Если зеркало имеет форму эллипсоида вращения, а свет распространяется от одного его фокуса Р к другому Q (причём путь без отражения невозможен), то оптическая длина пути луча PO’ + O’Q по свойствам эллипсоида равна всем остальным возможным, например PO» + О» Q; если на пути между теми же точками свет отражается от зеркала меньшей, чем у эллипсоида, кривизны (MM), реализуется минимальный путь, если же большей (зеркало NN) – максимальный. Условие экстремальности оптической длины пути сводится к требованию, чтобы была равна нулю вариация от интеграла

,

где А и В – точки, между которыми распространяется свет. Это выражение и представляет собой математическую формулировку принципа Ферма.

В волновой теории света принцип Ферма представляет собой предельный случай принципа Гюйгенса – Френеля и применим, когда можно пренебречь дифракцией света (когда длина световой волны достаточно мала по сравнению с характерными для задачи размерами): рассматривая лучи как нормали к волновым поверхностям, легко показать, что при всяком распространении света оптической длины их путей будут иметь экстремальные значения. Во всех случаях, когда необходимо учитывать дифракцию, принцип Ферма перестаёт быть применимым.

4.Преломоение света на плоской границе раздела 2-х сред. Полное внутреннее отражение

Если световой пучок падает на поверхность, разделяющую две прозрачные среды разной оптической плотности, например воздух и воду, то часть света отражается от этой поверхности, а другая часть — проникает во вторую среду. При переходе из одной среды в другую луч света изменяет направление на границе этих сред. Это явление называется преломлением света.

Законы преломления света.

Из всего сказанного заключаем:


1 . На границе раздела двух сред различной оптической плотности луч света при переходе из одной среды в другую меняет своё направление.
2. При переходе луча света в среду с большей оптической плотностью угол преломления меньше угла падения; при переходе луча света из оптически более плотной среды в среду менее плотную угол преломления больше угла падения.
Преломление света сопровождается отражением, причём с увеличением угла падения яркость отражённого пучка возрастает, а преломлённого ослабевает. Это можно увидеть проводя опыт, изображённом на рисунке. Следовательно, отражённый пучок уносит с собой тем больше   световой   энергии,   чем   больше   угол   падения.

Пусть MN -граница раздела двух про зрачных сред, например, воздуха и воды, АО-падающий луч, ОВ — преломленный луч, -угол падения, -угол преломления, -скорость распространения света в первой среде, — скорость распространения света во второй среде .

Первый закон преломления звучит так: отношение синуса угла падения к синусу угла преломления является постоянной величиной для данных двух сред:

, где — относительный показатель преломления (показатель преломления второй среды относительно первой).

Второй закон преломления света очень напоминает второй закон отражения света:

падающий луч, луч преломленный и перпендикуляр, проведенный в точку падения луча, лежит в одной плоскости.

Полное внутреннее отражение

Наблюдается для электромагнитных или звуковых волн на границе раздела двух сред, когда волна падает из среды с меньшей скоростью распространения (в случае световых лучей это соответствует бо́льшему показателю преломления).

С увеличением угла падения , угол преломления также возрастает, при этом интенсивность отражённого луча растет, а преломленного — падает (их сумма равна интенсивности падающего луча). При некотором критическом значении интенсивность преломленного луча становится равной нулю и происходит полное отражение света. Значение критического угла падения можно найти, положив в законе преломления угол преломления равным 90°:

5. Призмы

Призма — оптический элемент из прозрачного материала (например, оптического стекла) в форме геометрического тела — призмы, имеющий плоские полированные грани, через которые входит и выходит свет. Свет в призме преломляется. Важнейшей характеристикой призмы является показатель преломления материала, из которого она изготовлена. Виды призм: Дисперсионные призмы. Отражательные призмы. Поляризационные призмы.

Дисперсионные призмы Дисперсионные призмы используют в спектральных приборах для пространственного разделения излучений различных длин волн.

Отражательные призмы Отражательные призмы используют для изменения хода лучей, изменения направления оптической оси, изменения направления линии визирования, для уменьшения габаритных размеров приборов. Классифицируются отражательные призмы по нескольким признакам:


  • количеству отражений в призме

  • наличию или отсутствию «крыши»

  • характеру конструкции призмы

  • углу излома оптической оси

Также, особую нишу среди отражательных призм занимают составные призмы, — состоящие из нескольких частей, разделённых воздушными промежутками. Некоторые широко распространённые призмы получили собственные имена.

  • Призма Аббе

  • Призма Аббе-Порро

6. Тонкие линзы. Формула тонкой линзы

Линзой называется прозрачное тело, ограниченное двумя сферическими поверхностями. Если толщина самой линзы мала по сравнению с радиусами кривизны сферических поверхностей, то линзу называют тонкой. Линзы входят в состав практически всех оптических приборов. Линзы бывают собирающими и рассеивающими. Собирающая линза в середине толще, чем у краев, рассеивающая линза, наоборот, в средней части тоньше Линзы входят в состав практически всех оптических устройств. Линзы (Рис.3) делятся на собирающие и рассеивающие

Схема тонкой линзы

Рис.3,Собирающие (a) и рассеивающие (b) линзы и их условные обозначения.

Главной оптической осью линзы считается ось, прожодящая через центры кривизны её поверхностей. В тонкой линзе точки пересечения главной оптической оси с обеими поверхностями линзы сливаются в одну точку О.(Т.к. очень большие радиусы кривизны приближаются к плоскостям, то сферические поверхности теоретически сливаються в одну плоскость ). Эта точка называется оптическим центром линзы. Тонкая линза имеет одну главную плоскость, которая общая для двух сферических поверхностей и проходит через центр призмы и перпендикулярна к главной оптической оси. Все прямые, проходящие через оптический центр линзы, называются побочными оптическими осями линзы. Важным является то, что все лучи, идущие через оптический центр линзы, не преломляются.

Поток монохроматических параллельных лучей или пучков лучей с осями их узких конусов , нормалльных к сферической границе раздела (к главной плоскости, называют парксиальными (приосевыми) пучками. При этом, пройдя через неё сходятся в главном фокусе линзы F2. Главные фокусы линзы лежат на главной оптической оси линзы. Точки, расположенные на главной оптической оси линзы с двух сторон оптического центра на равных расстояниях f2., называются главными фокусами линзы . Плоскости, проходящие через главные фокусы f2 линзы и перпендикулярные к её главной оптической оси, называются фокальными плоскостями линзы .

Формула тонкой линзы.

Формула тонкой линзы связывает между; собой три величины: расстояние от предмета до линзы d, расстояние от линзы до изображения f и фокус ное расстояние линзы F:

В формуле тонкой линзы фокусное расстояние ОF обозначается буквой F. Если линза собирающая, то > 0, если линза рассеивающая, то перед ставится знак «минус». Если изображение действительное, то > 0; если изображение воображаемое, то перед ставиться знак «минус». Все величины в формулу линзы подставляются в метрах.

7. Построение изображений в линзах

Опыт показывает, что параксиальные лучи света, выходящие из одной светящейся точки, после прохождения через линзу сходятся также в одной точке, которая является изображением светящейся точки. Поэтому для построения изображения точки достаточно взять два любых луча, но лучше те, ход которых после преломления заранее известен: 1 — луч, идущий через оптический центр; 2 — луч, параллельный главной оптической оси; 3 — луч, проходящий через передний фокус собирающей линзы (или продолжение луча 3 проходит через задний фокус рассеивающей линзы) (рис. 16.41).

Рис. 16.41

Положение изображения действительного предмета и егоразмеры зависят от положения предмета относительно линзы. Пусть d — расстояние от предмета до линзы, f — расстояние от линзы до изображения. Построим изображение плоского предмета АВ, расположенного на различных расстояниях d от линзы. Если линза собирающая, то при d>2F (рис. 16.42) изображение действительное, перевернутое, уменьшенное,F

При F  (рис. 16.43) изображение действительное, перевернутое, увеличенное, f>2F.


Рис. 16.43

При d (рис. 16.44) изображение мнимое, прямое, увеличенное, находится с той же стороны от линзы, что и сам предмет, но дальше предмета (f>d).


Рис. 16.44

В рассеивающей линзе (рис. 16.45) изображение действительного предмета всегда мнимое, прямое, уменьшенное, находится между линзой и ее фокусом со стороны изображаемого предмета. 

8.Глаз как оптический прибор. Лупа, Микроскоп, фотоаппарат.

Глаз. Основным источником зрения является глазное яблоко, за зрачком находится хрусталик, а сзади сетчатка. Оптическую роль в глазе выполняет элемент, имеющий форму двояковыпуклой линзы и наз-ся хрусталиком. К краям хрусталика прикреплены мышцы, которые сжимают или растягивают хрусталик, в результате меняются радиусы кривизны сферич. пов-ти хрусталика и соответственно фокусные расстояния. При изменении расстояния d до наблюдаемого объекта, расстояние f от хрусталика до сетчатки остается неизменным, а меняется фокусное расстояние. Недостатки зрения – близорукость и дальнозоркость.

Лупой называют собирающую тонкую линзу с малым фокусным расстоянием (5-10 см).увеличение лупы: , расстояние наилучшего зрения.

Закон отражения света • Джеймс Трефил, энциклопедия «Двести законов мироздания»

Представьте, что вы направили тонкий луч света на отражающую поверхность, — например, посветили лазерной указкой на зеркало или полированную металлическую поверхность. Луч отразится от такой поверхности и будет распространяться дальше в определенном направлении. Угол между перпендикуляром к поверхности (нормалью) и исходным лучом называется углом падения, а угол между нормалью и отраженным лучом — углом отражения. Закон отражения гласит, что угол падения равен углу отражения. Это полностью соответствует тому, что нам подсказывает интуиция. Луч, падающий почти параллельно поверхности, лишь слегка коснется ее и, отразившись под тупым углом, продолжит свой путь по низкой траектории, расположенной близко к поверхности. Луч, падающий почти отвесно, с другой стороны, отразится под острым углом, и направление отраженного луча будет близким к направлению падающего луча, как того и требует закон.

Закон отражения, как любой закон природы, был получен на основании наблюдений и опытов. Можно его вывести и теоретически — формально он является следствием принципа Ферма (но это не отменяет значимости его экспериментального обоснования).

Ключевым моментом в этом законе является то, что углы отсчитываются от перпендикуляра к поверхности в точке падения луча. Для плоской поверхности, например, плоского зеркала, это не столь важно, поскольку перпендикуляр к ней направлен одинаково во всех точках. Параллельно сфокусированный световой сигнал — например, свет автомобильной фары или прожектора, — можно рассматривать как плотный пучок параллельных лучей света. Если такой пучок отразится от плоской поверхности, все отраженные лучи в пучке отразятся под одним углом и останутся параллельными. Вот почему прямое зеркало не искажает ваш визуальный образ.

Однако имеются и кривые зеркала. Различные геометрические конфигурации поверхностей зеркал по-разному изменяют отраженный образ и позволяют добиваться различных полезных эффектов. Главное вогнутое зеркало телескопа-рефлектора позволяет сфокусировать в окуляре свет от далеких космических объектов. Выгнутое зеркало заднего вида автомобиля позволяет расширить угол обзора. А кривые зеркала в комнате смеха позволяют от души повеселиться, разглядывая причудливо искаженные отражения самих себя.

Закону отражения подчиняется не только свет. Любые электромагнитные волны — радио, СВЧ, рентгеновские лучи и т. п. — ведут себя в точности так же. Вот почему, например, и огромные принимающие антенны радиотелескопов, и тарелки спутникового телевидения имеют форму вогнутого зеркала — в них используется всё тот же принцип фокусировки поступающих параллельных лучей в точку.

Вывод законов отражения и преломления света из принципа Ферма

Принцип наименьшего времени Ферма можно грубо сформулировать как: «Путь, пройденный луч света между двумя точками — это путь, который можно пройти в наименьшее время » (из Википедии). Ферма использовал это наблюдение, чтобы получить законы отражения и преломления света.

Отражение света

Закон отражения света гласит, что угол, под которым падает луч, на отражающей поверхности равен углу, под которым она отражается от поверхность.2}} $

$ \ подразумевает \ sin {{\ theta} _i} = \ sin {{\ theta} _r} $

$ \ подразумевает {\ theta} _i = {\ theta} _r \, \ blacksquare $

$ $

Преломление

Закон преломления света или закон Снеллиуса гласит, что отношение синусы углов падения и преломления равны обратной соотношение показателей преломления двух сред.

Со ссылкой на изображение выше, закон Снеллиуса математически можно выразить как,

$ \ frac {\ sin {\ theta} _i} {\ sin {\ theta} _r} = \ frac {n_2} {n_1} $

Теперь докажем это с помощью принципа Ферма.

Мы знаем,

$ \ text {показатель преломления среды} = \ frac {\ text {скорость света в вакууме}} {\ text {скорость света в среде}} $

Пусть скорость света в наших средах будет $ v_1 = \ frac {c} {n_1} $ и $ v_2 = \ frac {c} {n_2} $. 2}} $

$ \ подразумевает n_1 \ sin {\ theta} _i = n_2 \ sin {\ theta} _r $

$ \ implies \ frac {\ sin {\ theta} _i} {\ sin {\ theta} _r} = \ frac {n_2} {n_1} \, \ blacksquare $

Что есть… вариационный принцип?

Вариационные принципы играют фундаментальную роль в большей части математической физики и являются ключевой темой в моих собственных исследованиях. Об этом много, поэтому давайте начнем с небольшого рассказа.

1. Раненая корова и законы физики

Утром жарким летним днем ​​фермер заметил, что одна из его коров сломала ногу в поле. Несчастный зверь не мог двигаться долгое время. Чтобы не допустить обезвоживания коровы, фермер должен был принести ей воду из ручья, граничащего с полем.Пока фермер пошел за ведром, он подумал, как лучше всего выполнить эту задачу. Какой самый короткий маршрут он мог бы выбрать, сначала он идет к ручью, а затем к корове?

Совершенно очевидно, что фермер должен пройти по прямой к реке, а затем по другой прямой к корове. Все мы в школе учили, что прямая линия — это кратчайший путь между двумя точками. Но есть еще много способов объединить две прямые в подходящий путь.В какую точку на берегу реки следует пойти фермеру, чтобы тропа была как можно короче?

Если вы в течение минуты поиграете с разными возможностями, вы сможете догадаться, что обе линии должны образовывать одинаковый угол с берегом реки. Это простое условие, когда два угла равны, — это все, что нужно для определения кратчайшего пути.

Кратчайший путь к корове через реку состоит из двух прямых линий, расположенных под одинаковым углом к ​​берегу реки.

Корова со сломанной ногой — это плохо, но могло быть и хуже. Что, если корова упала в реку? Он не сможет вернуться на сушу со сломанной ногой. К счастью, река не слишком глубокая, поэтому неуклюжее животное не утонет, но фермеру придется перейти в реку, чтобы помочь ему.

А как бы фермерам быстрее всего добраться до коровы? Кратчайший путь будет прямой, но можно с уверенностью предположить, что фермер может бежать по полю быстрее, чем через ручей.Поэтому стоит выбрать более длинный путь, если более короткий его участок находится в воде. Самый быстрый маршрут может выглядеть так:

Самый короткий путь — прямая линия, но самый быстрый имеет изгиб.

Проблемы, с которыми сталкивается наш фермер, являются примерами вариационных проблем. Мы стремимся минимизировать некоторое количество (расстояние или время, пройденное в этих примерах). Если мы нашли оптимальное решение, то любое небольшое изменение этого решения будет немного хуже. Это дает нам первое объяснение имени вариационная задача .

Коровы физики

Почему мы заботимся об этих проблемах? Разве не будет никакой разницы, если фермеру потребуется на несколько секунд больше, чтобы добраться до коровы, не так ли? И выбор более длинного маршрута, вероятно, тратит меньше времени, чем чрезмерное обдумывание ситуации. Так в чем же вся суета?

Оказывается, физика очень похожа на фермер, пытающийся помочь своей корове.

В качестве первого примера рассмотрим луч света, отражающийся в зеркале. Из всех возможных путей от источника света через зеркало до места, где заканчивается луч света, он будет кратчайшим.Это потому, что закон отражения гласит, что падающий и отраженный лучи будут находиться под одним углом к ​​зеркалу. И, как выяснил наш фермер, равные углы создают кратчайший путь.

Или все наоборот? Можно сказать, что закон отражения имеет , потому что свет всегда проходит кратчайший путь.

А что насчет коровы в реке? Что ж, не только фермер медленнее движется в воде, но и свет. Он движется со «скоростью света» (около 300.000 км / с) в вакууме, немного медленнее в воздухе и намного медленнее в таких материалах, как вода или стекло. Это важно, потому что я солгал вам раньше: свет не обязательно идет по кратчайшему пути, он идет по самому быстрому пути . Если бы скорость света была везде одинаковой, это не имело бы никакого значения. Но если присутствуют разные материалы, то скорость зависит от того, где вы находитесь. Так, если, например, луч света попадает в воду с воздуха, он делает резкий поворот. Точно так же, как наш фермер достиг своей водной коровы.

Входящий ( «падающий» ) луч света может отражаться под тем же углом или преломляться под углом, определяемым законом Снеллиуса. В обоих случаях угол таков, что свет достигает места назначения как можно быстрее. [Изображение Nilok на wikimedia commons]

Явление, когда свет меняет направление при попадании в другую среду, называется преломлением . Вы могли выучить формулу для угла преломления (известную как закон Снеллиуса ) на уроках физики в средней школе:

\ (\ Displaystyle n_i \ грех \ theta_i = n_R \ sin \ theta_R.\)

Но вам не нужно понимать эту формулу, потому что она просто отражает тот факт (не каламбур), что свет идет самым быстрым путем. Если вы хотите провести расчеты, вам понадобятся формулы. Но если вы хотите понять, что происходит , вариационный принцип еще лучше. Этот конкретный вариационный принцип, согласно которому свет всегда движется по наиболее быстрому пути, называется принципом Ферма .

Как мы увидим ниже, свет не исключение. Многие другие физические системы описываются вариационными принципами.Они являются краеугольным камнем каждой части современной физики. Как и многие «законы» физики, закон отражения и закон Снеллиуса являются не чем иным, как следствием простого вариационного принципа.

2. Вариационные принципы в статике

Рассмотрите идиллический пейзаж холмов…

Нет, подожди. Сотрите это! Представьте себе эти идеализированные одномерные холмы:

Функция U дает высоту U (x) ландшафта в позиции x.

Места, где мяч не может сразу начать катиться с холма, — это те места, где касательная линия к холму горизонтальна: вершины холмов и низы долин.С точки зрения исчисления, это значения \ (x \), где производная от \ (U \) равна нулю:

\ (\ Displaystyle \ гидроразрыва {\ mathrm {d} U (x)} {\ mathrm {d} x} = 0 \)

Это приводит нас ко второй интерпретации слова вариационный . Производная — это бесконечно малая скорость изменения функции. У нас может быть только минимум или максимум, если эта скорость изменения, эта вариация , равна нулю. Вариационные задачи ищут ситуацию, когда бесконечно малое изменение некоторой величины равно нулю.

В нашем одномерном ландшафте таких локаций восемь. Есть восемь состояний равновесия , в которых мяч будет оставаться в покое, если на него не действует внешняя сила:

Между оранжевыми и синими шарами есть явная разница. Оранжевые — на вершинах холмов. Каждый из них находится в локальном максимуме функции \ (U \). Это приводит к печальным последствиям: как только мяч немного сдвинется в любую сторону, он начнет катиться с холма в сторону от равновесия.Мы называем эти виды равновесия нестабильным . Синие шары, с другой стороны, находятся в стабильных положениях равновесия. Если синий шар получит небольшой удар, он будет колебаться, пытаясь достичь равновесия, но в конце концов он вернется на прежнее место.

Другими словами, вариационный принцип

\ (\ Displaystyle \ гидроразрыва {\ mathrm {d} U (x)} {\ mathrm {d} x} = 0 \)

определяет все равновесия, но если мы хотим убедиться, что у нас есть стабильное равновесие, нам нужно дополнительное условие.2 x}> 0. \)

Оба условия вместе гарантируют, что \ (U \) имеет локальный минимум в \ (x \), или что мяч будет в устойчивом равновесии в положении \ (x \).

Функция \ (U \) называется потенциалом системы. В этом случае, когда сила тяжести является единственной задействованной силой, потенциал — это, по сути, высота. В более сложных системах потенциал будет более сложной функцией переменных системы, но его использование останется прежним. Равновесия находятся путем применения вариационного принципа к потенциалу.Устойчивые положения равновесия — это локальные минимумы потенциала.

3. Вариационные принципы в динамике

Нахождение равновесия системы — это еще не все. Хорошо знать, где система может находиться в состоянии покоя, но часто мы также хотим понять, как она движется, когда она не находится в состоянии покоя. Каким-то чудом это тоже регулируется вариационными принципами.

Предположим, мы хотим отслеживать мяч, катящийся по нашему одномерному ландшафту.

Обозначим положение мяча в момент \ (t \) через \ (x (t) \).Мы можем построить график положения во времени, так что \ (x (t) \) начертит кривую на \ ((x, t) \) — плоскости. Большинство таких кривых невозможно реализовать, если мяч движется только под действием силы тяжести. Те, которые могут быть, называются решениями системы. Для каждого начального положения и скорости мяча будет ровно одно решение. Но как найти решение?

Является ли какая-либо из этих кривых решением? Как мы можем сказать, на каком графике будет отображаться положение мяча?

Наиболее распространенный подход — использовать второй закон Ньютона: сила равна массе, умноженной на ускорение.2} = \ frac {F (x)} {m}. \)

Это дифференциальное уравнение (второго порядка) . Если начальное положение \ (x (0) \) и начальная скорость \ (\ frac {\ mathrm {d} x (t)} {\ mathrm {d} t} \ Big | _ {t = 0} \) даны, то можно решить определить \ (x (t) \) для всех значений времени \ (t \). (По крайней мере, теоретически. Только для относительно простых функций \ (F (x) \) можно будет записать это решение как красивую формулу для вычисления \ (x (t) \).)

Вместо второго закона Ньютона мы снова можем использовать вариационный принцип.2 — U (x (t)) \ right) \ mathrm {d} t, \)

, где квадратные скобки \ ([x] \) указывают, что \ (S \) зависит от функции \ (x \) в целом, а не только от конкретного значения \ (x (t) \).

Мы ищем минимизаторы \ (S \) в следующем смысле. Пусть начальная позиция (в момент времени \ (0 \)) будет \ (a \), а конечная позиция (в момент времени \ (T \)) \ (b \), то есть \ (x (0) = a \) и \ (x (T) = b \). Тогда \ (x \) является решением, если \ (S [x] \) меньше, чем \ (S [y] \) для любой другой функции \ (y \) с такими же граничными значениями \ (y (0) = а \) и \ (у (Т) = Ь \).

Путем хитроумных вычислений, включающих вариации функции \ (x \), можно увидеть, что функции, минимизирующие \ (S \), в точности удовлетворяют второму закону Ньютона. И снова знаменитый закон физики оказывается следствием вариационного принципа.

Вариационные принципы широко распространены в физике. Я рассмотрел здесь только простые примеры, но оказалось, что почти всю современную физику можно сформулировать с помощью вариационных принципов.Фактически, самый простой способ описать физическую теорию — это часто записывать то, что она сводит к минимуму.

Сохраненное количество

На этом история не заканчивается. Вместо того, чтобы смотреть на функции с фиксированными граничными значениями для получения второго закона Ньютона, мы могли бы смотреть только на функции, удовлетворяющие второму закону Ньютона, но оставлять граничные значения неопределенными. Затем аналогичные «умные вычисления» дают некоторую информацию о граничных значениях. Более конкретно, они производят сохраняемых величин , таких как энергия системы, которые принимают то же значение в конечный момент времени, что и в начальный момент времени (и действительно в любой промежуточный момент).

Какие именно сохраняющиеся величины получаются в результате этой процедуры, зависит от симметрии системы. Теорема Нётер, названная в честь математика начала 20-го века Эмми Нётер, утверждает, что каждой симметрии соответствует сохраняющаяся величина. Например, если система инвариантна относительно сдвига (например, бильярдные шары катятся по плоскости), то ее полный импульс сохраняется, а если она инвариантна относительно вращения (например, планеты, вращающиеся вокруг Солнца), то угловой момент сохраняется.

Знание сохраняемых количеств системы помогает понять ее динамику на многих уровнях. Если вы ищете точное решение, численное приближение или качественное понимание поведения, сохраняемые величины всегда будут полезны. И если вы читали «Что такое… интегрируемая система?», Вы знаете, что они являются ключом к области очень своеобразных динамических систем.

5. Источники и дополнительная литература

Как и в случае со многими концепциями, связанными с физикой, хорошее место для начала чтения — это лекции Фейнмана: некоторые соответствующие главы — «Оптика: принцип наименьшего времени» и «Принцип наименьшего действия».

Несмотря на то, что эти физические представления (и математика) не изменились с тех пор, как лекции Фейнмана были опубликованы более полувека назад, передовые технологии научного общения продвинулись вперед. В настоящее время есть отличные обучающие видео на подобные темы.

В большинстве вводных текстов по классической механике вариационным принципам не уделяется должного внимания. Заметное исключение (и отличная книга) —

.
  • Леви, Марк. Классическая механика с вариационным исчислением и оптимальным управлением: интуитивное введение .American Mathematical Soc., 2014.

Пример фермера и коровы основан на задаче

.
  • Станкова, Звезделина и Том Рике, ред. Десятилетие математического кружка Беркли: американский опыт, том II . American Mathematical Soc., 2015.

Произошла ошибка при настройке вашего пользовательского файла cookie

Этот сайт использует файлы cookie для повышения производительности. Если ваш браузер не принимает файлы cookie, вы не можете просматривать этот сайт.


Настройка вашего браузера для приема файлов cookie

Существует множество причин, по которым cookie не может быть установлен правильно. Ниже приведены наиболее частые причины:

  • В вашем браузере отключены файлы cookie. Вам необходимо сбросить настройки своего браузера, чтобы он принимал файлы cookie, или чтобы спросить вас, хотите ли вы принимать файлы cookie.
  • Ваш браузер спрашивает вас, хотите ли вы принимать файлы cookie, и вы отказались. Чтобы принять файлы cookie с этого сайта, нажмите кнопку «Назад» и примите файлы cookie.
  • Ваш браузер не поддерживает файлы cookie. Если вы подозреваете это, попробуйте другой браузер.
  • Дата на вашем компьютере в прошлом. Если часы вашего компьютера показывают дату до 1 января 1970 г., браузер автоматически забудет файл cookie. Чтобы исправить это, установите правильное время и дату на своем компьютере.
  • Вы установили приложение, которое отслеживает или блокирует установку файлов cookie. Вы должны отключить приложение при входе в систему или проконсультироваться с системным администратором.

Почему этому сайту требуются файлы cookie?

Этот сайт использует файлы cookie для повышения производительности, запоминая, что вы вошли в систему, когда переходите со страницы на страницу. Чтобы предоставить доступ без файлов cookie потребует, чтобы сайт создавал новый сеанс для каждой посещаемой страницы, что замедляет работу системы до неприемлемого уровня.


Что сохраняется в файле cookie?

Этот сайт не хранит ничего, кроме автоматически сгенерированного идентификатора сеанса в cookie; никакая другая информация не фиксируется.

Как правило, в файлах cookie может храниться только информация, которую вы предоставляете, или выбор, который вы делаете при посещении веб-сайта. Например, сайт не может определить ваше имя электронной почты, пока вы не введете его. Разрешение веб-сайту создавать файлы cookie не дает этому или любому другому сайту доступа к остальной части вашего компьютера, и только сайт, который создал файл cookie, может его прочитать.

Отражение и преломление

Отражение и преломление

Отражение и преломление

Принцип Ферма:

Уравнения Максвелла можно использовать для вывода законов отражения и преломления, которые говорят нам, как световые волны ведут себя на границе между двумя среды с разными показателями преломления.В 1650 г. Ферма открыл способ объяснить отражение и преломление как следствие одного единственного принципа. Это называется принципом наименьшего time или Принцип Ферма .

Предположим, мы хотим, чтобы свет попадал из точки A в точку B с учетом некоторой границы. состояние. Например, мы хотим, чтобы свет отражался от зеркала или пройти через кусок стекла на пути из пункта А в пункт Б. Принцип Ферма заявляет, что из всех возможных путей, по которым может идти свет, которые удовлетворяют граничные условия, свет идет по пути, который требует минимум времени.

Модуль 7: Вопрос 1

Рассмотрим двух человек у океана.

  • В ситуации Человек 1 на пляже, а человек 2 в воде. Человек 1 должен связаться с человеком 2 как можно быстрее. (Спасатель может добраться до попавшего в беду пловца.) Какой путь должен человек 1 дубль? Приведите качественные аргументы. Предположим, что человек 1 может бежать быстрее на пляже, чем купаться в океане.
  • В ситуации B человек 1 и человек 2 оба находятся на пляже.Человек 1 должен добраться до человека 2 как можно быстрее, пока впервые попадает в океан воды. (Может быть, нужно потушить пожар.) Какой путь следует человек 1 взять? Приведите качественные аргументы.

Обсудите это со своими однокурсниками на дискуссионном форуме!
Рассмотрим скорость человека 1 на песке и в воде.
Какой путь занимает меньше всего времени в ситуации A и в ситуации B.

Свет может двигаться быстрее по воздуху, чем по воде.Качественные аргументы, которые вы приводите о пути, который занимает меньше всего времени у человека, находящегося рядом с Граница раздела песок-вода в ситуациях A и B также применима к пути света попадание из воздуха в воду или отражение от границы раздела воздух-вода. Это принцип Ферма. Принцип Ферма автоматически приводит к принципу обратимости лучей в геометрической оптике. Неважно, едете ли вы из пункта А в пункт Б или От B к A путь, который занимает наименьшее количество времени, тот же.


Отражение

Отражение — это резкое изменение направления распространения волны. который устанавливает границу между двумя разными медиа. По крайней мере, некоторая часть приходящая волна остается в той же среде. Предположим, что падающий луч света угол θ i к нормали к плоскости касательная к границе. Затем отраженный луч образует угол θ r с этой нормалью и лежит в той же плоскость как падающий луч, так и нормаль.

Закон отражения:
θ i = θ r .

Примечание. В геометрической оптике углы всегда измеряются относительно нормали к границе раздела.

Зеркальное отражение происходит на гладких плоских границах. Тогда Касательная плоскость к границе — это сама граница. Отражение при грубом, нерегулярные границы диффузное отражение . Гладкая поверхность зеркала зеркально отражает свет, а шероховатая поверхность стены диффузно отражает свет.
Коэффициент отражения поверхностного материала равен доля энергии падающей волны, которая отражается от поверхности. В коэффициент отражения зеркала близок к 1.

Проблема:

Сколько раз будет показан падающий луч, показанный на рисунке? справа быть отражается каждым из параллельных зеркал?

Решение:

  • Рассуждение:
    После каждого пути между зеркалами луч проходит расстояние d в рост.
  • Детали расчета:
    Имеем d / (1 м) = tan (5 o ), d = tan (5 o ) m = 8,75 см. В поэтому луч должен пройти 11 раз между зеркалами, чтобы набрать высоту. 1 м, 6 раз влево и 5 раз вправо.

Преломление

Преломление — это изменение направления распространения волны, когда волна переходит из одной среды в другую и меняет свою скорость.Световые волны преломляется при переходе границы из одной прозрачной среды в другую потому что скорость света в разных средах разная. Предположим, что свет волны сталкиваются с плоской поверхностью куска стекла после первоначального движения через воздух, как показано на рисунке справа.

Что происходит с волнами, когда они переходят в стекло и продолжают движение Через стекло?
Скорость света в стакане или воде меньше скорости света в вакууме или воздухе.Скорость света в данном веществе равна v = c / n, где n — показатель преломления вещества. Типичные значения показателя преломления стекла: от 1,5 до 1,6, поэтому скорость света в стекле составляет примерно две трети скорость света в воздухе. Следовательно, расстояние между волновыми фронтами будет в стекле короче, чем в воздухе, так как волны проходят меньшее расстояние за период T.
Если f — частота волны, а T = 1 / f — период, т.е.е. временной интервал между последовательными гребнями, проходящими фиксированный точка в пространстве, тогда λ 1 = v 1 T = cT / n 1 и λ 2 = v 2 T = cT / n 2 , или

λ 1 / λ 2 = n 2 / n 1 .

Теперь рассмотреть волновые фронты и соответствующие им световые лучи, приближающиеся к поверхности под углом.
Мы видим, что лучи будут изгибаться, когда волна переходит от воздуха к стеклу.В изгиб происходит потому, что волновые фронты не проходят так далеко за один цикл в стекло, как в воздухе. Как видно на диаграмме, фронт волны на полпути к стекло проходит меньшее расстояние в стекле, чем в воздухе, в результате чего оно изгиб посередине. Таким образом, луч, перпендикулярный фронту волны, также изгибы. Ситуация похожа на марширующий оркестр, идущий по грязному полю в угол к краю поля. Ряды изгибаются от скорости марширующих. уменьшено грязью.

Величина отклонения света зависит от угла падения и на показатели преломления стекла и воздуха, определяющие изменение скорость. Из рисунка видно, что λ 1 / λ 2 = sinθ 1 / sinθ 2 .
Но λ 1 / λ 2 = n 2 / n 1 . Следовательно, n 2 / n 1 = sinθ 1 / sinθ 2 , или n 1 sinθ 1 = n 2 sinθ 2 .

Это закон Снеллиуса , или закон преломления .
n 1 sinθ 1 = n 2 sinθ 2 .

Когда свет переходит из одной прозрачной среды в другую, лучи наклоняться к нормали к поверхности, если скорость света меньше во второй среде чем в первом. Лучи отклоняются от этой нормы, если скорость света во второй среде больше, чем в первой.Картинка на справа показывает световая волна падает на стеклянную пластину.
Одна часть волны отражается, а другая часть преломляется как он переходит в стекло. Лучи наклоняются к нормали. На втором Граница раздела между стеклом и воздухом свет, попадающий в воздух, снова преломляется. Теперь лучи отклоняются от нормали.

Параллельное смещение: когда луч света проходит из воздуха через прямоугольный блок стекла и угол падения не равен нулю, то при выходе из стекла луч света смещается параллельно его исходный путь.

Проблема:

Путь света в воздухе, падающего на стекло и проходящего через него пластина показана на рисунке справа. Угол падающего луча к нормали составляет 45 o и равен углу отраженный луч. Пройденный луч преломляется под углом 28 o к лучу. нормали и выходит из стекла под углом 45 o к нормали, угол равен к падающему лучу. Какой показатель преломления у стекла?

Решение:

  • Рассуждение:
    Воспользуемся законом Снеллиуса.
    n i sinθ i = n t sinθ t
  • Детали расчета:
    Как луч попадает из воздуха в стакан, имеем n i = 1, θ i = 45 o , а θ t = 28 o . Следовательно, мы имеем n t = n i sinθ / sinθ t = sin45 o / sin28 o = 1,5.
Проблема:

Предположим, что световой сигнал проходит по прямой линии через среду с индексом преломления n = 1.55, например оптическое волокно. Сколько времени это занимает сигнал проехать 20 см?

Решение:

  • Рассуждение:
    Скорость света в среде v = c / n.
  • Детали расчета:
    v = d / t. t = d / v = (0,2 м) * 1,55 / (3 * 10 8 м / с) = 1,03 * 10 -9 с = 1,03 нс.

На границе между двумя прозрачная среда, свет частично отражается и частично преломляется. Отношение отраженной интенсивности к падающей интенсивности называется коэффициент отражения R и отношение передаваемой интенсивности к падающей интенсивность называется коэффициентом пропускания T.Энергосбережение требует, чтобы R + T = 1 (если нет поглощения).


Солнечный свет

Солнечный свет исходит от внешней поверхности Солнца в области, называемой фотосфера. Эта область имеет температуру ~ 5800 o C. распределение длин волн в солнечном свете определяется температурой фотосфера. Не весь солнечный свет виден. ЭМ волны в инфракрасном и ультрафиолетовая часть электромагнитного спектра также производится в фотосфере.

Солнечный свет движется от Солнца к Земле через пустое пространство со скоростью света c. Когда он входит в атмосферу Земли, он преломляется. Индекс преломление воздуха около уровня моря составляет всего 1.0003, поэтому преломление едва достигает заметно. Молекулы воздуха, молекулы воды и пыль также рассеивают часть свет ( рэлеевское рассеяние ). Эти частицы рассеивают более коротковолновый свет более эффективно, чем более длинноволновый. Рассеяние на крошечных частицах всегда наиболее эффективно, когда Длина волны ЭМ примерно соответствует размеру крошечной частицы.Размеры молекул и пылевых частиц намного меньше, чем у длины волн видимого света, поэтому синий свет с самой короткой длиной волны обеспечивает наилучшее соответствие. Синий свет рассеивается больше, чем красный. Самый солнечный свет попадает прямо в наши глаза, но рассеянный свет достигает нас более сложный путь со всех сторон. Таким образом, мы видим блестящие желтый диск солнца (прямой свет) и довольно равномерно голубое небо (рассеянный свет).

Когда солнце встает или садится, свет должен проходить большое расстояние через Атмосфера Земли, чтобы достичь наших глаз. Большая часть синего света рассеивается во время этого прохождения через атмосферу и прямой свет от солнца появляется красный. Дополнительные частицы пыли и золы от загрязнения, лесных пожаров или извержения вулканов усиливают рэлеевское рассеяние и ответственны за необычно красные восходы и закаты.

Облака и туман состоят из относительно крупных капель воды размером более длины волн видимого света.Все длины волн видимого спектра равны очень эффективно рассеиваются этими крупными частицами, так что очень мало прямых солнечный свет достигает наших глаз. Однако рассеянный свет не имеет определенного цвета, а облака и туман кажутся белыми.

Проблема:

Предположим, есть облако, состоящее из каких-то неизвестных частиц, которые поглощают а не рассеивать видимое излучение. Какого цвета было бы это облако появляются днем?

Решение:

  • Рассуждение:
    Облако будет казаться черным, поскольку видимый свет не достигнет наблюдатель со стороны облака.

Дополнительная информация:

В Класс физики: преломление и лучевая модель света, уроки 1 и 2

Если вы пропускаете регулярные лекции, обратите внимание на эту видеолекцию.

Лекция 29: Закон Снеллиуса, преломление и полное отражение

Преломление света — Science Learning Hub

Преломление — это искривление света (это также происходит со звуком, водой и другими волнами), когда он переходит от одного прозрачного вещества к другому.

Это изгибание за счет преломления позволяет нам получать линзы, увеличительные стекла, призмы и радуги. Даже наши глаза зависят от этого отклонения света. Без преломления мы не смогли бы фокусировать свет на сетчатке.

Изменение скорости вызывает изменение направления

Свет преломляется всякий раз, когда он проходит под углом в вещество с другим показателем преломления (оптической плотностью).

Это изменение направления вызвано изменением скорости.Например, когда свет перемещается из воздуха в воду, он замедляется, заставляя его продолжать двигаться под другим углом или в другом направлении.

Насколько отклоняется свет?

Величина изгиба зависит от двух вещей:

  • Изменение скорости — если вещество заставляет свет ускоряться или замедляться больше, он будет больше преломляться (изгибаться).
  • Угол падающего луча — если свет входит в вещество под большим углом, величина преломления также будет более заметной.С другой стороны, если свет проникает в новое вещество прямо (под углом 90 ° к поверхности), свет все равно будет замедляться, но совершенно не изменит направление.

Показатель преломления некоторых прозрачных веществ

Вещество

Показатель преломления

Скорость света в веществе
с

Угол преломления, если
падающий луч входит в
вещество под углом 20º

Воздух

1.00

300

20

Вода

1,33

226

14.9885

2 905

14.9885 905 905

13,2

Алмаз

2,4

125

8.2

Все углы измеряются от воображаемой линии, проведенной под углом 90 ° к поверхности двух веществ. Эта линия изображена пунктирной линией и называется нормалью.

Если свет попадает в какое-либо вещество с показателем преломления выше (например, из воздуха в стекло), он замедляется. Свет изгибается на по направлению к нормальной линии.

Если свет проникает внутрь вещества с показателем преломления ниже (например, из воды в воздух), он ускоряется.Свет отклоняется на от нормальной линии на .

Более высокий показатель преломления показывает, что свет замедляется и сильнее меняет направление по мере попадания в вещество.

Линзы

Линза — это просто изогнутый блок из стекла или пластика. Есть два вида линз.

Двояковыпуклая линза в середине толще, чем по краям. Такой тип линзы используется для увеличительного стекла. Параллельные лучи света можно сфокусировать в точку фокусировки. Двояковыпуклая линза называется собирающей линзой.

Двояковогнутая линза в середине тоньше, чем по краям. Световые лучи преломляются наружу (расходятся) при входе в линзу и снова при выходе.

Преломление может создать спектр

Исаак Ньютон провел знаменитый эксперимент, используя треугольный стеклянный блок, называемый призмой. Он использовал солнечный свет, проникающий через его окно, чтобы создать спектр цветов на противоположной стороне своей комнаты.

Этот эксперимент показал, что белый свет состоит из всех цветов радуги.Эти семь цветов запоминаются аббревиатурой ROY G BIV — красный, оранжевый, желтый, зеленый, синий, индиго и фиолетовый.

Ньютон показал, что каждый из этих цветов не может быть превращен в другие цвета. Он также показал, что их можно рекомбинировать, чтобы снова получить белый свет.

Объяснение разделения цветов заключается в том, что свет состоит из волн. Красный свет имеет большую длину волны, чем фиолетовый. Показатель преломления красного света в стекле немного отличается от показателя преломления фиолетового света.Фиолетовый свет замедляется даже больше, чем красный свет, поэтому он преломляется под немного большим углом.

Показатель преломления красного света в стекле равен 1,513. Показатель преломления фиолетового света 1,532. Этой небольшой разницы достаточно для того, чтобы свет с более короткой длиной волны преломлялся сильнее.

Rainbows

Радуга возникает из-за того, что каждый цвет преломляется под немного разными углами, когда входит, отражается изнутри и затем оставляет каждую крошечную каплю дождя.

Радугу легко создать с помощью пульверизатора и солнечного света.Центром круга радуги всегда будет тень вашей головы на земле.

Вторичная радуга, которую иногда можно увидеть, возникает из-за того, что каждый луч света дважды отражается от внутренней стороны каждой капли, прежде чем она улетит. Это второе отражение приводит к тому, что цвета вторичной радуги меняются местами. Красный цвет находится вверху основной радуги, но во вторичной радуге красный цвет находится внизу.

Идеи упражнений

Используйте эти упражнения со своими учениками для дальнейшего изучения рефракции:

  • Исследование рефракции и подводной охоты — учащиеся направляют копья на модель рыбы в емкости с водой.Когда они направляют свои копья к рыбе, они промахиваются!
  • Калькулятор угла преломления Задача — учащиеся выбирают два типа прозрачного вещества. Затем они вводят угол падающего луча в калькулятор электронной таблицы, и для них рассчитывается угол преломленного луча.
  • Свет и зрение: правда или ложь? — учащиеся участвуют в интерактивном мероприятии «правда или ложь», в котором освещаются распространенные альтернативные представления о свете и зрении. Это задание можно выполнять индивидуально, парами или всем классом.

Полезные ссылки

Узнайте больше о различных видах радуги и о том, как они образуются, на веб-сайте Atoptics — «Отражение радуги» и «Приказы радуги».

Узнайте больше о человеческих линзах, оптике, фоторецепторах и нервных путях, обеспечивающих зрение, из этого учебного пособия из Biology Online.

00000nam a22000615u 45009780735421578AIP Publishing20210428103624.0cr mn ||| mna || 200902t2020 |||| nyu |||| gs 0 || 0 | eng d9780735421288paperback9780735421295epub9780735421301epdf10.1063/9780735421578doiSCI055000bisacshSCI040000bisacshPereyra, Николас А. Действительные и комплексные числа для физиков Николас А. Перейра Мелвилл, Нью-Йорк Интернет-ресурс AIP Publishing2020 (274 страницы) texttxtrdacontentcomputercrdamediaonline Inendory Числа — Логические числа в натуральных числах — Дополнительные свойства целых чисел — Неравенства в целых числах — Рациональные числа — Дополнительные свойства рациональных чисел — Неравенства в рациональных числах — Действительные числа — Дополнительные свойства действительных чисел — Неравенства в действительных числах — Комплексные числа.Действительные и комплексные числа для физиков представляет собой подробное и строгое введение в натуральные, целые, рациональные и действительные числа. Он устраняет пробел в математической библиотеке и предлагает своим читателям прочную основу для аналитики и решения проблем. В книге рассматриваются: — Углубленное введение в действительные числа, предназначенное для физиков и естествоиспытателей — Различные системы счисления без отхода от теоретических дискуссий — Конкретные знания и навыки, основанные на исчислении, применительно к физике Ценный ресурс для ученых, работающих с Настоящие числа, эта книга также помогает преподавателям, преподающим теорию чисел и естественные науки, и укрепит у студентов математические навыки и навыки решения задач.Общая физика / GeneralbisacshМатематическая физика / Mathematical & ComputationalbisacshПечатное издание 9780735421288https: //doi.org/10.1063/978073542157800000nam a22000615u 45009780735421608AIP Publishing20210428103640.0cr mn || 0 | eng d9780735421585paperback9780735421615epub9780735421592epdf10.1063 / 9780735421608doiTEC008090bisacshLevi, A. F. J. Важнейший перенос электронов для физики устройств FJ LeviMelville, New York AIP Publishing2020онлайн-ресурс (266 страниц) texttxtrdacontentcomputercrdmediaonline resourcecrrdacarrierПолупроводники и квантованные состояния — Плотность состояний и статистика частиц — Пропускание электронов — Оптимальная конструкция гетероструктурного устройства — Полуклассическое электронное равновесие — Перенос электронов — Электронное полупроводниковое рассеяние — Электронное полупроводниковое рассеяние -Тип полупроводников. Транспорт неравновесных неосновных носителей. К квантовой инженерии.Essential Electron Transport for Device Physics представляет ключевые элементы электронного транспорта, наиболее применимые к изучению полупроводниковых электронных устройств. Это удобный справочник и краткое изложение фундаментальных знаний, которые необходимо понять, прежде чем исследовать более сложные модели и концепции электронных устройств. Содержание служит основой для ученых и инженеров без необходимости инвестировать в специализированные подробные исследования.TECH & ENGR./Electronics / Semiconductorsbisacsh Печатное издание 9780735421585 https://doiorg / 10.1063 / 978073542160800000nam a22000615u 45009780735421646AIP Publishing20210428103652.0cr mn ||| mna || 201016t2020 |||| nyu |||| gs 0 || 0 | eng d9780735421622paperback9780735421653epub9780735421639epdf10.1063 / 9780735421646doiSCI096000bisacshКисачанин, БраниславF = ma Конкурсы 2011–2019 Руководство по решениям Бранислав Кисачанин; Эрик К. ЧжанМелвилл, Нью-Йорк Интернет-ресурс AIP Publishing2020 (324 страницы) texttxtrdacontentcomputercrdamediaonline resourcecrrdacarrierQuestions — Вопросы — Вопросы — Вопросы — Вопросы — Вопросы — Вопросы — Вопросы для экзамена A — Вопросы для экзамена B — Вопросы для экзамена A — Вопросы для экзамена B — Ресурсы — Дополнительные ресурсы.В этой книге представлены четкие и подробные решения всех задач ежегодного конкурса F = ma, проводимого Американской ассоциацией учителей физики (AAPT) в период с 2011 по 2019 год. Конкурс является предшественником USAPhO (Национальной олимпиады США по физике) и IPhO (Международная физическая олимпиада). F = ma Contests: 2011-2019 Solutions Manual устраняет значительный пробел в существующей литературе по подготовке к соревнованиям и является отличным ресурсом для студентов, готовящихся к экзаменам по физике и олимпиадам.Science / Mechanics / SolidsbisacshZhang, Eric K. Печатное издание 9780735421622 https://doi.org/10.1063/978073542164600000nam a22000615u 45009780735421769AIP Publishing20210428103710.0cr mn ||| mna || 210223| 0 | eng d9780735421776paperback9780735421752epub9780735421745epdf10.1063 / 9780735421769doi2020938690SCI040000bisacshSCI009000bisacshMarro, ХоакинФазовые переходы в сером веществе Архитектура мозга и динамика разума Хоакин Марро; Хоакин Дж. Торрес, Мелвилл, Нью-Йорк, AIP Publishing, 2021, Интернет-ресурс (170 страниц) texttxtrdacontentcomputercrdamediaonline resourcecrrdacarrierMind Atoms — Критичность, сложность и смежная динамика — Формирование отношений — Выявление критического — The Brain In Silico — Завершение бестиария.Фазовые переходы в серой материи: архитектура мозга и динамика разума связывает сложные системы, которые мы знаем как «разум» и «мозг», с простыми физическими понятиями, такими как «фазовый переход» и «критичность», и устанавливает между ними тесную математическую связь. Серьезный обзор актуальных вопросов науки — от взаимодействия и корреляции до эмерджентности, масштабной инвариантности, аттракторов, шума и хаоса — эта книга демонстрирует их актуальность для интеллекта и сознания. Результатом является значительный и полезный портрет того, что «разум» в настоящее время означает для науки, который объединяет широко разбросанные, а иногда и трудно найти темы в одном ресурсе.Основные моменты: — Обеспечивает последовательную основу для существования критичности в мозге и показывает, как можно понять некоторые из его основных выдающихся свойств — Глубоко исследует предположение о том, что фазовый переход является наиболее подходящей концепцией для понимания разума. — Предлагает прочную основу для тех, кто заинтересован в создании полного и полезного портрета мозга. Эта книга предназначена для продвинутых аспирантов и аспирантов в области физики, прикладной математики, биологии и медицины. Это отличный ресурс для практиков и тех, кто интересуется последними достижениями нейробиологии.Наука / Физика / Математика и вычисления | gs 0 || 0 | eng d9780735421813paperback9780735421806epub9780735421783epdf10.1063 / 9780735421790doiSCI085000bisacshLi, MinghengAnalysis and Design of Membrane Processes, A Системный подход Мингхенг ЛиМелвилл, Нью-Йорк, AIP Publishing2020онлайн-ресурс (288 страниц) texttxtrdacontentcomputercrdmediaonline resourcecrrdacarrierIntroduction — Трехмерный CFD-анализ гидродинамики и массопереноса в заполненных спейсерами каналах обратного осмоса — Квазидвумерное прогнозное моделирование на основе данных модуля RO Оптимизация удельного энергопотребления в системе обратного осмоса морской воды — Оптимизация и проверка оборудования для работы BWRO — Систематический анализ и оптимизация осмоса с замедленным давлением для выработки электроэнергии — Гибридный RO-PRO для энергоэффективного опреснения — Пакетная работа RO и PRO.Анализ и проектирование мембранного процесса: системный подход подчеркивает основы и новые технологии в области промышленного опреснения обратным осмосом и мембранных процессов. Он обеспечивает уникальную перспективу системной инженерии для работы мембран, уделяя особое внимание анализу, проектированию и оптимизации мембранных процессов. Объяснение математических и оптимизационных знаний вводится и затем применяется на протяжении всей книги. Ключевые темы включают: — Гидродинамика и массоперенос в мембранах обратного осмоса (RO) — Прогностические модели для работы модуля обратного осмоса — Анализ и оптимизация обратного опреснения солоноватой и морской воды — Производство энергии с помощью осмоса с замедленным давлением (PRO) — Интеграция RO и PRO для энергоэффективного опреснения — Динамическая работа пакетного RO и пакетного PRO Эта работа является важным чтением для исследователей, интересующихся мембранными процессами, и тех, кто работает в водном хозяйстве, а также студентов и аспирантов.Science / Mechanics / Fluidsbisacsh Печатное издание 9780735421813https: //doi.org/10.1063/97807354217

00nam a22000615u 45009780735421820AIP Publishing20210429102025.0cr mn ||| mna || 210424t2021 |||| nyu |||| nyu |||| 0 | eng d9780735421851 paperback9780735421844epub9780735421837epdf10.1063 / 9780735421820doi2020938777SCI024000bisacshBright, Jamie M. Синтетическое солнечное излучение Моделирование солнечных данных Джейми М. Брайт: под редакцией Джейми М. Брайт Мелвилл, Нью-Йорк Интернет-ресурс AIP Publishing2021 (226 страниц) texttxtrdacontentcomputercrdmediaonline resourcecrrdacarrierIntroduction to Synthetic Solar Irradiance — — Principles and Key Applications: Principles and Applications of Synthetic Solar Irradiance — — Established to Synthetic Solar Irradiance — — Established to Synthetic Solar Irradiance — — Established Генерация данных энергетической освещенности — — Подтверждение синтетических данных солнечной освещенности — — Промышленное применение синтетической освещенности: тематическое исследование солнечной энергии — — Будущее синтетической солнечной освещенности -.Synthetic Solar Irradiance: Modeling Solar Data — первая книга, в которой рассматриваются принципы и методы этой развивающейся области. Заполняя пробел в литературе, эта своевременная книга редактируется одним из ведущих мировых авторитетов в области искусственного солнечного излучения при участии других ведущих экспертов. Он охватывает ключевые приложения синтетического солнечного излучения и установленные математические подходы для производства синтетических временных рядов. Другие ключевые темы включают: — Примеры использования ключевых определений, литературы и доступности данных — Определение успеха генерируемого синтетического излучения — Проблемы и альтернативы, стоящие перед синтетическим солнечным излучением Эта книга идеальна для чтения для исследователей солнечной энергетики, инженеров-энергетиков и инженеров-электриков, прикладных математиков, прикладных компьютерных специалистов и специалистов в области оценки солнечных ресурсов и солнечных ферм — или для всех, кто работает с колебаниями мощности, вводимой в сеть от солнечных панелей. .Science / EnergybisacshПечатное издание 9780735421851https: //doi.org/10.1063/978073542182000000nam a22000615u 45009780735421882AIP Publishing20210428103747.0cr mn ||| mna || 200928t2020 |||| nyu |||| gs ||| 0 | анг d9780735421899paperback9780735421875epub9780735421868epdf10.1063 / 9780735421882doiSCI009000bisacshSCI051000bisacshMeli, Джером A.Physics радиационной дозиметрии, TheJerome А. MeliMelville, Новый YorkAIP Publishing2020online ресурс (272 страниц) texttxtrdacontentcomputercrdamediaonline resourcecrrdacarrierBasic понятия — взаимодействие Вероятность и сечение взаимодействия — взаимодействия заряженных частиц: Эластичный кулоновское рассеяние — Тормозная способность среды для заряженных частиц — Диапазон заряженных частиц — Доза в пучках заряженных частиц — Взаимодействие фотонов: кинематика — Взаимодействие фотонов: сечения — Производство рентгеновских лучей — Доза в пучках фотонов — Измерение кермы воздуха, Экспозиция и доза в рентгеновских пучках кВ — Теория резонатора — Измерение дозы в пучках МВ — Ионизационная камера — Дозиметры, отличные от ионных камер — Взаимодействие и дозиметрия нейтронов.Физика радиационной дозиметрии представляет основы радиационной дозиметрии через логическую последовательность тем. В этом всестороннем изложении излагаются базовые концепции для новичка, а затем на основе этой основы вводятся темы, которые необходимо знать всем учащимся, такие как взаимодействия заряженных частиц и материи и взаимодействия фотонов и материи. Эта книга предлагает: — Подробное, основанное на физике объяснение представленных тем — Главы, содержащие наборы задач для студентов — Легко следовать вводной радиационной физике — Углубленные обсуждения и выводы Физика радиационной дозиметрии идеально подходит для аспирантов по медицинской физике и является бесценным ресурсом для резидентов и практикующих врачей-физиков.НАУКА / Науки о жизни / Биофизика 0 | eng d9780735421943paperback9780735421950epub9780735421974epdf10.1063 / 9780735421967doiSCI055000bisacshВеннинг, Карл Дж. Преподавание физики в средней школе Природа преподавания физики Карл Дж. Веннинг; Ребекка Э. Вейра, Мелвилл, Нью-Йорк, AIP Publishing2020онлайн-ресурс (286 страниц) texttxtrdacontentcomputercrdamediaonline resourcecrrdacarrierСтановление учителем физики в старшей школе — Научная грамотность — Преподавание и обучение — Научная эпистемология — Научная грамотность — Введение в физику — Научная практика и интеллектуальные навыки — Исследование низкой сложности — Исследование средней и высокой сложности — Математические методы исследования — Преподавание и исследования в области физического образования.Обучение физике в средней школе основано на том принципе, что учителя должны быть образованы, а не обучены, и помогает сформировать существенную и прочную основу для нового способа обучения. Предлагая сочетание теории и практики, эти книги описывают более 40 важных тем и поощряют исследовательский подход к преподаванию физики. Они включают многочисленные примеры и полезные ресурсы. Природа преподавания физики: — Исследует роль образовательных исследований, философии преподавания и научной эпистемологии как основы хорошего преподавания — Делает сильный акцент на обучении через исследование — Готовит учителей с прочной философской и практической базой, уделяя особое внимание характер преподавания физики Преподавание физики в средней школе подходит кандидатам в учителя, опытным учителям физики, работающим без отрыва от производства, и педагогам, которым требуется подробное справочное руководство по многим предметам, рассматриваемым в сегодняшних движениях за реформу преподавания.НАУКА / Общая физикаbisacshVieyra, Rebecca E. Печатное издание 9780735421943https: //doi.org/10.1063/978073542196700000nam a22000615u 45009780735422018AIP Publishing20210428103814.0cr mn |||| mna20 |||| n20 ||| 0 | eng d9780735421981 paperback9780735421998epub9780735422001epdf10.1063 / 9780735422018doiSCI055000bisacshВеннинг, Карл Дж. Преподавание физики в средней школе Взаимодействие со студентами-физиками Карл Дж. Веннинг; Ребекка Э. Вейра Мельвилл, Нью-Йорк, AIP Publishing2020онлайн-ресурс (300 страниц) texttxtrdacontentcomputercrdamediaonline resourcecrrdacarrierМинимизация сопротивления студентов запросам — Хорошие педагогические практики — Эффективное использование технологий — Активное и заинтересованное обучение — Совместное обучение — Обучение студентов NGSS и альтернативные реалии Решение проблем — Метапознание и саморегуляция — Дифференцированное обучение — Культурное, языковое и гендерное разнообразие — Учащиеся с особыми потребностями — Социальное и эмоциональное обучение — Содействие академическому успеху — Беседы в классе и сократический диалог — Доска.Обучение физике в средней школе основано на том принципе, что учителя должны быть образованы, а не обучены, и помогает сформировать существенную и прочную основу для нового способа обучения. Предлагая сочетание теории и практики, эти книги описывают более 40 важных тем и поощряют исследовательский подход к преподаванию физики. Они включают многочисленные примеры и полезные ресурсы. Взаимодействие со студентами-физиками: обращается к практическим методам поддержки обучения студентов в повседневном классе Подробно о том, как способствовать активному участию и сотрудничеству на справедливой основе, управлять трудностями в обучении и дифференциациями, а также другими ситуациями, с которыми учителя сталкиваются в классе. прочная философская и практическая основа, ориентированная на взаимодействие со студентами, преподающими физику в средней школе, подходит для кандидатов в учителя, опытных учителей физики без отрыва от производства и преподавателей, которым требуется подробное справочное руководство по многим предметам, рассматриваемым в сегодняшних движениях за реформу преподавания.НАУКА / Общая физикаbisacshVieyra, Rebecca E. Печатное издание 9780735421981 https://doi.org/10.1063/978073542201800000nam a22000615u 45009780735422056AIP Publishing20210428103825.0cr mn |||| n20 ||| 0 | eng d9780735422025paperback9780735422049epub9780735422032epdf10.1063 / 9780735422056doiSCI055000bisacshВеннинг, Карл Дж. Преподавание физики в средней школе Управление классом физики Карл Дж. Веннинг; Ребекка Э. Вейра Мелвилл, Нью-Йорк, AIP Publishing2020онлайн-ресурс (270 страниц) texttxtrdacontentcomputercrdamediaonline resourcecrrdacarrierУправление классом — Юридическое, безопасное и этичное обучение — Разработка учебной программы — Планирование подразделения — Дизайн обучения — Простая оценка — Комплексная оценка — Оценка учеников — Старшая школа Поиск своей первой преподавательской работы — Первый год преподавания и последующий период — Профессиональная практика и рост.Обучение физике в средней школе основано на том принципе, что учителя должны быть образованы, а не обучены, и помогает сформировать существенную и прочную основу для нового способа обучения. Предлагая сочетание теории и практики, эти книги описывают более 40 важных тем и поощряют исследовательский подход к преподаванию физики. Они включают многочисленные примеры и полезные ресурсы. Природа преподавания физики: — Исследует роль образовательных исследований, философии преподавания и научной эпистемологии как основы хорошего преподавания — Делает сильный акцент на обучении через исследование — Готовит учителей с прочной философской и практической базой с упором на характер преподавания физики Преподавание физики в средней школе подходит кандидатам в учителя, опытным учителям физики, работающим без отрыва от производства, и педагогам, которым требуется подробное справочное руководство по многим предметам, рассматриваемым в сегодняшних движениях за реформу преподавания.НАУКА / Общая физикаbisacshVieyra, Rebecca E. Печатное издание 9780735422025https: //doi.org/10.1063/978073542205600000nam a22000615u 45009780735422179AIP Publishing20210428103835.0cr mn ||| 0 | eng d9780735422148paperback9780735422162epub9780735422155epdf10.1063 / 9780735422179doiEDU029030bisacshSCI053000bisacshНельсон, Джейн БрейПреподавание геометрической оптики: студенческое издание Джейн Брей Нельсон; Джим Нельсон Мелвилл, Интернет-ресурс AIP Publishing2020 в Нью-Йорке (280 страниц) texttxtrdacontentcomputercrdamediaonline resourcecrrdacarrier Задание 1: История общества слепых — Задание 2: Источники света — Задание 3: Свойства света — «Изображение» точечной дыры — Задание 4: Тени и — Задание 5: Фазы Луны: A — Задание 6: Использование радужных очков для исследования взаимосвязи между освещенностью и расстоянием — Задание 7: Зависимость освещенности от расстояния (версия Texas Instruments CBL) — Задание 8: Освещенность в зависимости от расстояния (Вернье) — Задание 9: Практические задачи по теням и освещению — Задание 10: Изготовление парафинового фотометра и измерение силы света Солнца — Задание 11: Краткий исторический обзор скорости света — Задание 12: Как измерить угол с помощью транспортира — Задание 13: Объекты, плоские зеркала и свойства изображений — Задание 14: Отражение света от плоского зеркала — Задание 15: Отражение света от нескольких плоских зеркал — A Задание 16: Практические задания на плоских зеркалах (Версия A) — Задание 17: Практические задания на плоских зеркалах (Версия B) — Задание 18: Демонстрации с плоскими зеркалами — Задание 19A: Анализ иллюзии мираскопа — Задание 19B: Свойства сформированных изображений с помощью вогнутого зеркала — Задание 20: Свойства изображений, образованных вогнутыми / выпуклыми зеркалами — Задание 21: Семейная физика — Вогнутое зеркало — Задание 22: Рисование диаграмм лучей для изогнутых зеркал — Задание 23: Рабочий лист 1: Отражение и зеркала — Задание 24: Демонстрации с изогнутыми зеркалами — Задание 25: Преломление света, проникающего в жидкости — Задание 26: Демонстрация волшебного исчезающего раствора — Задание 27: Демонстрация аквариума — Задание 28: Преломление света через твердые тела — Задание 29: Показатель преломления, изменение длины волны и призмы — Задание 30: Критический угол и показатель преломления — Задание 31: Преломление света через волокна — Задание 32: Преломление и дифракция цвета — Задание 33: Относительное Индекс преломления — Задание 34: Свойства изображений, сформированных сходящейся линзой I — Задание 35: Свойства изображений, сформированных сходящейся линзой II — Задание 36: Глаз млекопитающих: применение системы линз — Задание 37: Свойства сформированного изображения с помощью расходящейся линзы I — Задание 38: Свойства изображения, формируемого расходящейся линзой II — Задание 39: Рисование лучевых диаграмм для линз — Задание 40: Рабочий лист 2: Преломление и линзы — Задание 41: Демонстрации расслаивающихся линз — Задание 42: Интересное задание Использование двух сходящихся линз — Задание 43: Практические задачи с линзами — Задание 44: Закон поглощения Бера – Ламберта — Задание 45: Эффективность различных лампочек — Задание 46: Радиометр.Преподавание геометрической оптики направляет учителей физики, чтобы помочь студентам развить основы понимания геометрической оптики. Краеугольный камень фотонных систем, геометрическая оптика, находит применение в широком спектре отраслей, включая технологии, медицину и военный сектор. В этой книге рассматриваются основы распространения, отражения и преломления света, а также использование простых оптических элементов, таких как зеркала, призмы, линзы и оптические волокна. Ключевые элементы включают в себя: — 46 занятий по геометрической оптике, охватывающих широкий круг тем — Простота реализации, с готовым листом для учеников и заметками для учителей — Ссылки на соответствующих преподавателей физики и преподавателей других наук в области научных стандартов нового поколения Эта книга станет бесценным ресурсом для обучения основам дисциплин.Преподавание геометрической оптики предназначено для вводных студентов колледжей и старших классов. ОБРАЗОВАНИЕ / Методы обучения и материалы / Наука и технологииbisacshSCIENCE / Физика / Оптика и свет mn ||| mna || 201029t2020 |||| ню |||| gs 0 || 0 | анг d9780735422216paperback9780735422193epub9780735422186epdf10.1063 / 9780735422209doiTEC021040bisacshChen, BoFundamentals из ресопряжения и Развязка техники в Твердотельный NMRBo ChenMelville, Новый YorkAIP Publishing2020online ресурс (358 страниц) texttxtrdacontentcomputercrdamediaonline resourcecrrdacarrierIntroduction ЯМР — Введение принципов ЯМР Метизы — Основы импульсного ЯМР и релаксации — The Матрица плотности и ее основные приложения в ЯМР — Тензоры и ЯМР с вращением под магическим углом — Основы методов восстановления связи при вращении под магическим углом — Основы методов разделения при вращении под магическим углом.Основы методов воссоединения и развязки в твердотельном ЯМР обеспечивают широкий охват, начиная с основ, за счет обновленных методов воссоединения и развязки. В нем подробно описаны пошаговые инструкции и показано, как твердотельный ЯМР с Magic Angle Spinning использует взаимодействие между механическим вращением и радиочастотными (RF) импульсами для активного включения (воссоединения) и выключения (разъединения) желаемых взаимодействий. Эта уникальная книга предлагает: — Охват от основных до сложных тем и включает в себя пошаговые выводы и упражнения — Концептуальные, экспериментальные и математические аспекты современного твердотельного ЯМР — Подробные инструкции по тензорному исчислению и пошаговые выводы для Помогите студентам научиться интегрировать фазу вращения под магическим углом Эта книга идеально подходит для студентов старших курсов, изучающих физику или родственную дисциплину, и практикующих физиков, заинтересованных в изучении этой темы.ТЕХ. & ENGR./Materials Sci / Тонкие пленки, поверхности и интерфейсы | 0 | анг d9780735422261paperback9780735422285epub9780735422278epdf10.1063 / 9780735422292doiTEC031020bisacshTEC031010bisacshOkedu, KennethEnhanced энергосистема Стабильность Использование Вдвойне-Fed Induction GeneratorsKenneth OkeduMelville, Новый YorkAIP Publishing2020online ресурс (280 страниц) texttxtrdacontentcomputercrdamediaonline resourcecrrdacarrierAugmentation ветра фермы Fault ехать через по DFIG на основе переменной скорости Ветрогенератор — Схема Новая защита DFIG — Повышение устойчивости с помощью прерывателя постоянного тока, лома и устройства FACTS — Преобразователи источника напряжения с регулируемым током и соображения сетевого кода — Стратегия гибридного управления для преобразователей энергии ветряных турбин с переменной скоростью — Влияние параметров модели трансмиссии — Использование преобразователей с параллельным чередованием и динамического Резисторы при трехфазных сбоях — Альтернативный источник напряжения Т-образный преобразователь на стороне сети — Преимущества и слабые места различных стратегий управления — Параллельно интегрированный конденсатор и модифицированный модулированный тормозной резистор — Дет Определение наиболее эффективного коммутирующего сигнала и положения тормозного резистора — синхронизация фильтра нижних частот контроллера угла наклона и схема демпфера, подключенная к сети.Повышенная стабильность энергосистемы с использованием индукционных генераторов с двойным питанием направлена ​​на новейшие схемы, моделирование и стратегии управления для улучшения ветряных турбин с регулируемой скоростью. В этой книге моделирование проводится с использованием современных пакетов программного обеспечения для моделирования различных типов симметричных и асимметричных разломов для анализа переходной устойчивости и методов моделирования для энергетических систем ветряных турбин. Ключевые особенности: — Обсуждаются индукционные генераторы с двойным питанием, устранение неисправностей и другие концепции повышения стабильности энергосетей — Рассматриваются характеристики сети — необходимое, но часто упускаемое из виду соображение в возобновляемых источниках энергии — Демонстрирует различные подходы и соответствующие стратегии управления который может быть очень эффективным для стабилизации ветряной электростанции, подключенной к сети. Инженеры и профессионалы, участвующие в строительстве и проектировании электрических сетей и ветряных электростанций, сочтут это ценным ресурсом.Эта книга также предназначена для профессоров и аспирантов в области электротехники, политиков и сторонников возобновляемых источников энергии. & ENGN. / Энергетические ресурсы / ElectricalbisacshTECH. & ENGN. / Power Resources / Alternative & RenwablebisacshПечатное издание 9780735422261https: //doi.org/10.1063/978073542229200000nam a22000615u 45009780735422414AIP Publishing20210428103915.0cr mn ||| mna || 201230t2020 |||| ny |||| 0 | eng d9780735422384paperback9780735422391epub9780735422407epdf10.1063/9780735422414doiSCI024000bisacshTEC031010bisacshTEC021040bisacshRen, JingzhengSoft-Matter тонкопленочные солнечные элементы Физические процессы и моделирование устройств Jingzheng Ren; Чжипэн Кан: отредактировал Цзинчжэн Рен; Zhipeng KanMelville, New York AIP Publishing2020онлайн-ресурс (280 страниц) texttxtrdacontentcomputercrdamediaonline resourcecrrdacarrierПроисхождение гистерезиса в перовскитных солнечных элементах — Процессы с переносчиками заряда и подвижные ионные солнечные динамические процессы в перовскитных солнечных элементах: прогресс и перспективы — Миграция ионов перовскитов в металлических элементах из галогенидов перовскитов. Элементы: идеи и перспективы — Рекомбинация заряда в органических солнечных элементах — Состояние переноса заряда и потери напряжения в органических солнечных элементах — Фотофизика в органических солнечных элементах — Физика устройств в органических солнечных элементах и ​​моделирование дрейфа-диффузии.Тонкопленочные солнечные элементы с мягким материалом: моделирование физических процессов и устройств представляет собой руководство по современным инновациям и разработкам в солнечных фотоэлектрических элементах. Отредактированная и написанная мировыми авторитетами в соответствующих областях, эта книга исследует недавно разработанные материалы и включает численные и вычислительные эксперименты в области исследования материалов. Он обеспечивает высший уровень исследования и использования нового класса фотоэлектрических материалов. Некоторые ключевые темы, затронутые в этой книге, включают: — Происхождение, теоретические исследования и моделирование устройств для перовскитных солнечных элементов — Рекомбинация заряда, переходные состояния и потери энергии в органических солнечных элементах — Физика устройств в органических солнечных элементах и ​​моделирование дрейфовой диффузии Тонкопленочные солнечные элементы с мягким материалом: моделирование физических процессов и устройств актуально для исследователей, докторантов, аспирантов и студентов старших курсов.Материаловеды и инженеры в секторе солнечной энергетики, политики и аналитики, а также другие отраслевые эксперты также найдут эту книгу бесценным ресурсом. НАУКА / EnergybisacshTECHNOLOGY & ENGINEERING / Power Resources / Альтернативные и возобновляемые источники & ИнтерфейсыbisacshKan, Zhipeng Печатное издание 9780735422384 https://doi.org/10.1063/978073542241400000nam a22000615u 45009780735422452AIP Publishing20210428103926.0cr mn ||| mna || 201212t2020 |||| ню |||| gs 0 || 0 | анг d9780735422421paperback9780735422438epub9780735422445epdf10.1063 / 9780735422452doiTEC019000bisacshSCI053000bisacshTEC017000bisacshBarat, KenManual лазерной SafetyKen Барат: Под редакцией Кен BaratMelville, New YorkAIP Publishing2020online ресурса (176 страниц) texttxtrdacontentcomputercrdamediaonline resourcecrrdacarrierBuilding блоки лазерной безопасности — Вертикальная безопасности Макет, методы и использованию — Практическая ценность Доброй лазера Программное обеспечение для обеспечения безопасности — Лазерная безопасность и микроскоп — Лазерные аудиты: на что смотрят и к чему готовятся — Открывающие глаза на лазерных защитных очках — Аппаратное обеспечение и оптический стол: соображения безопасности на столе — Лазерная безопасность для очень мощных лазеров — Промышленный лазер: Самый универсальный станок 21 века — ресурсы.Эта книга представляет собой четкое и краткое руководство для ученых, занимающихся исследованиями и разработками, работающих с лазерами. В нем рассматриваются несколько методов использования лазеров и подходов к безопасности лазеров, которые не встречаются в других текстах, включая использование и подходы с вертикальным лучом, безопасность с мощными лазерами и отражательную способность различных материалов. Руководство по безопасности при работе с лазером предоставляет ресурсы, представляющие непосредственную ценность для пользователя лазера и специалиста по безопасности. Эта книга, охватывающая все классы лазеров, — важный инструмент для сотрудников по лазерной безопасности, который может предоставить членам команды. — Представлено в сжатой форме с учетом последних передовых практик, правил и стандартов — Написано ведущим автором и авторитетными специалистами в данной области Руководство по лазерной безопасности специально написано для ученых, занимающихся исследованиями и разработками в академических и промышленных кругах.Это ценный учебник для профессионалов, работающих в области лабораторной безопасности. & ENGN. / Lasers & PhotonicsbisacshSCIENCE / Физика / Optics & LightbisacshTECH. & ENGN. / Промышленное здравоохранение и безопасность 0 | eng d9780735422704paperback9780735422728epub9780735422711epdf10.1063 / 9780735422698doiTEC008090bisacshTEC021020bisacshSCI032000bisacshNarita, TetsuoХарактеризация дефектов и уровней мощности NaritaT для устройств GaNo; Тецу Качи: отредактировал Тецуо Нарита; Tetsu KachiMelville, New York AIP Publishing2020онлайн-ресурс (224 страницы) texttxtrdacontentcomputercrdamediaonline resourcecrrdacarrierIntroduction — Методы анализа глубоких уровней в GaN — Глубинные уровни в GaN — Структурные дефекты в GaN, легированном магнием Вся GaN пластина — многофотонная микроскопия — будущие задачи: дефекты в устройствах питания на основе GaN из-за производственных процессов.В этой книге основное внимание уделяется дефектам в GaN, основанным на самых современных внутренних свойствах материалов, и рассматриваются глубокие уровни и их аналитические методы в широкой запрещенной зоне GaN. Он демонстрирует наноразмерные структуры протяженных дефектов в GaN с помощью просвечивающей электронной микроскопии на атомном уровне. Идентификация их глубоких уровней и протяженных дефектных структур представлена ​​путем сравнения с опубликованными расчетами из первых принципов. В нем рассматриваются новые технологии определения характеристик дефектов с использованием атомно-зондовой томографии, синхротронной рентгеновской дифракционной топографии в масштабе пластины и фотолюминесценции с многофотонным возбуждением, что позволяет проводить разнонаправленную характеристику структурных дефектов.Читатели узнают: — Электрические воздействия дефектов в вертикальных силовых устройствах на основе GaN. — Пути к контролю дефектов в процессе производства электрических устройств на основе GaN — Современные методы анализа дефектов полупроводников и электронных материалов Определение характеристик дефектов и глубоких уровней для силовых устройств на основе GaN является идеальным справочным материалом для отраслевых материаловедов работает в полупроводниковых материалах и устройствах. Он также подходит для специалистов в области DLTS, TEM, APT, XRDT и фотолюминесценции с многофотонным возбуждением.ТЕХ. & ENGR. / Электроника / ПолупроводникиbisacshTECH. & ENGR. / Материаловедение / Electronic MaterialsbisacshSCIENCE / Электронные микроскопы и микроскопия | 0 | eng d9780735422735paperback9780735422759epub9780735422742epdf10.1063 / 9780735422766doiEDU029030bisacshSCI053000bisacshНельсон, Джейн Брей Преподавание геометрической оптики: Заметки учителя Джейн Брей Нельсон; Джим Нельсон Мелвилл, Нью-Йорк Интернет-ресурс AIP Publishing2020 (336 страниц) texttxtrdacontentcomputercrdamediaonline resourcecrrdacarrier Действие 1: История общества слепых — Задание 2: Источники света — Задание 3: Свойства света — Пинхол — Задание 4: Тени и прямолинейное распространение света — Задание 5: Фазы Луны: моделирование — Задание 6: Использование радужных очков для исследования взаимосвязи между освещенностью и расстоянием — Задание 7: Освещенность в зависимости от расстояния — Задание 8: Освещенность в зависимости от расстояния — Задание 9: Практические задачи по теням и освещенности — Задание 10: Изготовление парафинового фотометра и измерение силы света Солнца — Задание 11: Краткий исторический обзор скорости света — Задание 12: Как измерить угол с помощью транспортира — Задание 13: Объекты, плоские зеркала , и свойства изображений — Задание 14: Отражение света от плоского зеркала — Задание 15: Отражение света от нескольких плоских зеркал s — Задание 16: Практические задания на плоских зеркалах — Задание 17: Практические задания на плоских зеркалах — Задание 18: Заметки об отражении света — Задание 19A: Анализ иллюзии мироскопа — Задание 20: Свойства изображений, сформированных вогнутыми / выпуклыми зеркалами — Задание 21: Семейная физика — Вогнутое зеркало — Задание 22: Рисование диаграмм лучей для изогнутых зеркал — Задание 23: Рабочий лист 1: Отражение и зеркала — Задание 24: Демонстрации с изогнутыми зеркалами — Задание 25: Отражение света, проникающего в жидкости — Задание 26: Демонстрация волшебного исчезающего раствора — Задание 27: Демонстрация в аквариуме — Задание 28: Преломление света через твердые тела — Задание 29: Показатель преломления, изменение длины волны и призмы — Задание 30: Критический угол и показатель преломления — Задание 31: Преломление света через волокна — Задание 32: Преломление и дифракция цвета — Задание 33: Аппарат для обучения физике — Задание 34: Свойства изображений, формируемых сходящейся линзой I — Задание 35: Свойства изображений, формируемых сходящейся линзой II — Задание 36: Глаз млекопитающих: применение системы линз — Задание 37: Свойства изображения, формируемого расходящейся линзой I — Задание 38: Свойства изображения, формируемого расходящимися линзами Линза II — Задание 39: Рисование лучевых диаграмм для линз — Задание 40: Рабочий лист 2: Преломление и линзы — Задание 41: Демонстрации расслаивающихся линз — Задание 42: Интересное задание с использованием двух сходящихся линз — Задание 43: Практические задачи с линзами — Задание 44: Закон поглощения Бера – Ламберта — Задание 45: Эффективность различных лампочек — Задание 46: Радиометр.Преподавание геометрической оптики: Заметки учителя направляют учителей физики, чтобы помочь учащимся развить базовые представления о геометрической оптике. Краеугольный камень фотонных систем, геометрическая оптика, находит применение в широком спектре отраслей, включая технологии, медицину и военный сектор. В этой книге рассматриваются основы распространения, отражения и преломления света, а также использование простых оптических элементов, таких как зеркала, призмы, линзы и оптические волокна. Ключевые элементы включают в себя: — 46 упражнений по геометрической оптике, охватывающих широкий круг тем — Простота реализации, с готовой копией ведомости для учащихся и заметками для учителей — Ссылки на соответствующих преподавателей физики и преподавателей других наук в области научных стандартов нового поколения Эта книга будет бесценным ресурсом для обучения основам дисциплин.Этот том содержит только примечания для Преподавания геометрической оптики: студенческое издание. ОБРАЗОВАНИЕ / Методы обучения и материалы / Наука и технологииbisacshSCIENCE / Физика / Оптика и свет mn ||| mna || 210415t2021 |||| ню |||| gs 0 || 0 | eng d9780735422964paperback9780735422971epub9780735422988epdf10.1063 / 9780735422995doi2021930782TEC031020bisacshTEC031010bisacshOkedu, KennethOnshore Wind Farms Динамическая стабильность и приложения в производстве водорода Кеннет ОкедуМелвилл, Нью-Йорк, AIP Publishing2021онлайн-ресурс (240 страниц) texttxtrdacontentcomputercrdamediaonline resourcecrrdacarrier АНАЛИЗ СТАБИЛЬНОСТИ ВЕТРОВОГО ХОЗЯЙСТВА — ПРИМЕНЕНИЕ ВЕТРОВОГО ХОЗЯЙСТВА.«Береговые ветряные фермы: динамическая стабильность и применение в производстве водорода» предлагает актуальное обсуждение новых технологий и методов для строительства ветряных электростанций и модернизации существующих ветряных турбин. Эффективная работа ветряных электростанций, подключенных к сетям, представляет собой серьезную проблему для операторов сетей, в основном из-за временного характера ветровой энергии. В этой книге рассматриваются улучшения динамических характеристик сети в сочетании с другими источниками, включая ядерную энергию, гидроэнергетику и тепловые турбины.Он исследует влияние устройств и стратегий управления, которые потенциально улучшают улавливание энергии ветра и стабилизируют существующие ветряные турбины с фиксированной скоростью в ветряных электростанциях, подключенных к сети. Эта своевременная книга: — Включает новую технологию для улучшенных ветряных турбин. — Включает разработки для повышения устойчивости сетей, использующих энергию ветра — Охватывает такие области применения, как производство водородного топлива и топливные элементы постоянного тока Эта книга, имеющая отношение к исследователям и инженерам, работающим в области ветроэнергетики, также подходит для студентов, интересующихся развитием сетей или другими формами возобновляемой энергии.Он также представляет интерес для специалистов по планированию и анализу политики в области ветроэнергетики. ТЕХНОЛОГИЯ И ИНЖИНИРИНГ / Энергетические ресурсы / ElectricalbisacshTECHNOLOGY & ENGINEERING / Power Resources / Alternative & Renewablebisacsh Печатная версия 9780735422964https: //doi.org/10.1063/9780735422995000003 | | mna || 210515t2021 |||| ню |||| gs 0 || 0 | анг d9780735422919paperback9780735421677epub9780735421660epdf10.1063 / 9780735421684doi2020952362SCI024000bisacshTEC021020bisacshAhuja, RajeevNext-Generation Материалы для BatteriesRajeev Ahuja: Под редакцией Раджива AhujaMelville, Новый YorkAIP Publishing2021online ресурс (290 страниц) texttxtrdacontentcomputercrdamediaonline resourcecrrdacarrierIntroduction: Фон вычислительных и экспериментальных исследований для следующего поколения эффективных материалов для батарей — Вычислительные и экспериментальный Методы определения материалов для аккумуляторов — Органические аккумуляторы: путь к устойчивым технологиям хранения электроэнергии — Подходы к машинному обучению на основе данных для расширенного моделирования аккумуляторов — Анодные материалы на основе графена для Li и Na аккумуляторов — 2D анодные материалы без углерода для следующего поколения Батареи — Анодные материалы 2D на основе MXene для батарей нового поколения — Анодные материалы на основе гетероструктуры Ван-дер-Ваальса — Полимерные электролиты для литиевых аккумуляторных батарей — Suitabl e Электродные материалы для гибридных конденсаторов — Перспективы будущего и направление материалов для батарей следующего поколения.Материалы нового поколения для аккумуляторов объединяют множество различных областей аккумуляторных технологий в единый ресурс и обобщают основы материалов для аккумуляторов. В нем подробно описаны инструменты, используемые при исследовании материалов, и описаны некоторые из наиболее многообещающих последних разработок. Книга: — Сочетает теоретические и вычислительные методы с экспериментальными исследованиями батарей, демонстрируя при этом, как результаты из одной области могут поддержать усилия в другой. — Исследует материалы, в том числе Na-ионные аккумуляторы, как альтернативы широко используемым сегодня литий-ионным аккумуляторам. — Предоставляет перспективы и направления для материалов для аккумуляторов следующего поколения. Эта своевременная книга является идеальным источником для докторов наук.D. и постдокторанты в области материалов для аккумуляторов, суперконденсаторов, физики твердого тела и электрохимии. Эта книга станет бесценным справочником для исследователей, занимающихся производством аккумуляторов, бытовой электроники, гибридных транспортных средств и других отраслей, а также политиков и тех, кто интересуется устойчивой энергетикой. 9780735422919https: //doi.org/10.1063 / 978073542168400000nam a22000615u 45009780735423152AIP Publishing20210528112800.0cr mn ||| mna || 210522t2021 |||| nyu |||| gs 0 || 0 | eng d9780735423121paperback9780735423138epub9780735423145epdf10.1063 / 9780735423152doi2021931482TEC031010bisacshBUS070040bisacshSCI075000bisacshRen, JingzhengPhotovoltaic Sustainability; Чжипенг КанМелвилл, Нью-Йорк, AIP Publishing2021, онлайн-ресурс (184 страницы) texttxtrdacontentcomputercrdamediaonline resourcecrrdacarrierГлобальный прогресс, перспективы и проблемы устойчивости солнечной энергии Фотоэлектрические технологии: Quo Vadis? — Технико-экономический анализ осуществимости и нормированная стоимость солнечной фотоэлектрической энергии — анализ углеродного следа жизненного цикла солнечной фотоэлектрической энергии Технологии — Анализ жизненного цикла солнечных фотоэлектрических технологий — Жизненный цикл окружающей среды Оценка различных солнечных фотоэлектрических технологий — с ориентацией на устойчивое развитие Принятие решений по выбору места размещения солнечной фотоэлектрической установки — инновации, политика и Последствия для продвижения устойчивой солнечной фотоэлектрической отрасли от руководства Перспектива.Photovoltaic Sustainability and Management изучает фотоэлектрические (PV) технологии, которые широко используются для преобразования света в электрическую энергию. В то время как производство фотоэлектрической энергии является устойчивым и без выбросов, производство и размещение фотоэлектрических модули оказывают воздействие на окружающую среду, что требует рассмотрения профессионалами отрасли. Эта книга: — Обсуждаются технико-экономические аспекты, углеродный след и устойчивость жизненного цикла. характеристики фотоэлектрических технологий. — Сосредоточен на нетехнических факторах, которые существенно влиять на эффективность развертывания фотоэлектрических систем, включая проблемы, инновации, политики, воздействие на окружающую среду, выбор местоположения и управление.- Исследует фотоэлектрические технологии от жизненный цикл и перспективы устойчивости. Устойчивое развитие и управление фотоэлектрическими системами предназначены для академические исследователи, технические специалисты и консультанты, занимающиеся фотоэлектрическими исследованиями или работающие в фотоэлектрических системах технологии или возобновляемая энергетика. ТЕХНОЛОГИЯ И ИНЖИНИРИНГ / Энергетические ресурсы / Альтернативные и возобновляемые источники энергии / EnergybisacshБизнес и ЭКОНОМИКА / Отрасли / Энергетика / Общество и OrganizationbisacshSCIENCE / Философия и социальные аспекты / Междисциплинарная физикаbisacshKan, Zhipeng Печатное издание 9780735423121https: // doi.org / 10.1063 / 978073542315200000nam a22000615u 45009780735423077AIP Publishing20210521124236.0cr mn ||| mna || 210514t2021 |||| ню |||| gs 0 || 0 | анг d9780735423046paperback9780735423053epub9780735423060epdf10.1063 / 9780735423077doi2021930894SCI053000bisacshElliott, BarryOptical CommunicationBarry ElliottMelville, Новый YorkAIP Publishing2021online ресурс (216 страниц) texttxtrdacontentcomputercrdamediaonline resourcecrrdacarrierIntroduction — передача света и электромагнитного спектра — Поляризация — Законы отражения, преломление и дифракция — Манипулирование света: Линзы и Призмы — Создание и обнаружение света — Кодирование информации в световод — Световод: Оптическое волокно — Компоненты оптической связи — Тестирование оптических систем.Оптическая связь — это основополагающий справочник по принципам и применению оптики. Книга предлагает широкое понимание основ оптики, физики и математики, лежащих в основе передачи света, устройств, влияющих на прохождение света, и практического применения оптической теории в коммуникациях. Он охватывает теоретические и практические применения оптических технологий. В книге показано, как физика оптики находит применение в телекоммуникациях, оптической связи в свободном пространстве и других средах.Эта книга: — Предлагает практическое руководство по оптике и ее приложениям в коммуникациях, дополняющее существующие работы по ее теории. — Рассматривает работу международных органов по стандартизации, таких как ITU и ISO, по согласованию физики, лежащей в основе предмета, и доступности стандарта. продукты, которые могут реализовать современные высокоскоростные сети связи. — Предоставляет вводные концепции для новичков в этой области, а также студентов и профессионалов. Эта книга подходит для физиков, инженеров, техников и химиков.Студенты бакалавриата или любой специалист в области оптики, которому может потребоваться подготовка в соответствующих приложениях или раннее знакомство с этой областью, найдут эту книгу ценным справочником. НАУКА / Физика / Оптика и свет / Оптика и оптическая физика org / 10.1063 / 978073542307700000nam a22000615u 45009780735423190AIP Publishing20210602162509.0cr mn ||| mna || 210526t2021 |||| ню |||| gs 0 || 0 | eng d9780735423169paperback9780735423176epub9780735423183epdf10.1063/9780735423190doi2021932213SCI063000bisacshEDU029000bisacshGrimes, Алиса Почитание учителей как профессионалов Истории и пути роста в классе и карьере Алиса Граймс; Николь Шроде; Ребекка Стобер; Шеннон Вачовски, Мелвилл, Нью-Йорк, AIP Publishing2021, Интернет-ресурс (168 страниц) texttxtrdacontentcomputercrdamediaonline resourcecrrdacarrierСодействие сообществу и сотрудничеству — Повышение уверенности и профессионализма — Проведение исследований в области образования — Влияние на политику в области образования — Расширение прав и возможностей студентов — Стать агентом перемен.«Уважение к учителям как профессионалам: истории и пути роста в классе и карьере» — это руководство по удержанию и привлечению преподавателей, использующих методы исследования в области обучения в классе. Он знакомит с процессом, позволяющим стать опубликованным исследователем в области образования; принять и разработать творческие индуктивные учебные программы; расширять права и возможности студентов; и построение сообщества как пути к лидерству в сфере образования. Проведя целенаправленное рецензируемое исследование, эта работа показывает, как личные и педагогические изменения превратили классы в пространства, где учащиеся могут чувствовать себя уполномоченными и побуждать к научным концепциям.Сильные стороны этой книги включают в себя переплетение теории с практикой, признание образования как профессиональной области с использованием языка, который легко соединяется как с новичками, так и с опытными учителями. Другие ключевые особенности включают: — Доступный, повествовательный и анекдотический стиль письма — Рекомендации, которые помогут решить проблему низкого уровня удержания учителей и их удовлетворенности работой — Обсуждение воспитания у учителей чувства общности и подходов к построению таких сообществ. Эта книга предназначена для администраторов, исследователей, учителей и других специалистов в области образования.Это ценный ресурс для аспирантов в сфере образования, сторонников политики и родителей. 45009780735422377AIP Publishing20210730021230.0cr mn ||| mna || 210806t2021 |||| ню |||| gs 0 || 0 | анг d9780735422346paperback9780735422353epub9780735422360epdf10.1063 / 9780735422377doi2021934605SCI013100bisacshSCI013070bisacshWang, YunMultiscale Моделирование электрохимических реакций и ProcessesYun Ван: Под редакцией Yun WangMelville, New YorkAIP Publishing2021online ресурса (138 страниц) texttxtrdacontentcomputercrdamediaonline resourcecrrdacarrierTheory-эксперимент Gap — первых принципов расчетов для моделирования электрохимической реакции: Введение в методы и приложения — Численное моделирование границ раздела наэлектризованное твердое тело-жидкость — Теория и практика моделирования молекулярной динамики с постоянным потенциалом — Подходы среднего поля и модифицированные методы Пуассона-Больцмана для моделирования электрохимических систем накопления энергии — Многомасштабное моделирование процессов переноса заряда в органических полупроводниках.Многомасштабное моделирование электрохимических реакций и процессов — это практическое руководство по многомасштабным вычислительным методологиям. Он предлагает целостное понимание влияния условий реакции на общую производительность электролизеров, топливных элементов и устройств хранения энергии. В этой книге рассматриваются условия реакции, такие как электролит, приложенный потенциал смещения и тип опоры, а также то, как эти факторы определяют общую производительность устройств. Эти темы впервые освещены в одной книге.В этой книге представлены: — Всестороннее исследование разрыва между экспериментальной теорией электрохимических реакций. — Современные многомасштабные методы моделирования влияния реакционной среды, включая электролит, потенциал смещения и тип опоры, на эффективность преобразования энергии в электрохимических ячейках. — Обсуждение того, как применить эти методы многомасштабного моделирования к различным приложениям в технологиях чистой энергии. Многомасштабное моделирование электрохимических реакций и процессов служит ценным ресурсом для ученых, инженеров и студентов, интересующихся электрохимией, многомасштабным моделированием и приложениями чистой энергии.Это также ресурс для постоянно увеличивающегося числа доступных курсов по моделированию материалов. НАУКА / Химия / ЭлектрохимияbisacshSCIENCE / Химия / Вычислительное и молекулярное моделирование || 210807t2021 |||| ню |||| gs 0 || 0 | анг d9780735423329paperback9780735423336epub9780735423343epdf10.1063 / 9780735423350doi2021935657SCI022000bisacshSCI055000bisacshLekner, JohnElectrostatics проводящих цилиндров и SpheresJohn LeknerMelville, Новый YorkAIP Publishing2021online ресурс (240 страниц) texttxtrdacontentcomputercrdamediaonline resourcecrrdacarrierFundamentals — бисферические Координаты — две сферы во внешнем поле — Два Заряженный Spheres — Bicylindrical Координаты — Два цилиндра во внешнем Поле — два заряженных цилиндра — уединенный конечный цилиндр — суммы и интегралы.Написанная признанным экспертом в данной области, книга «Электростатика проводящих цилиндров и сфер» представляет теорию простым и физическим способом, начиная с основных принципов и переходя к деталям электростатики пар сфер и пар цилиндров. Также освещается непростая тема конечного заряженного цилиндра. В книге представлены точные решения, по возможности, в закрытой форме, включая недавние и новые результаты. Также представлены приложения к инженерным, физическим, химическим и биологическим проблемам.Ключевые темы включают: — Основы тензора поляризуемости и емкостных коэффициентов сборки проводников. — Строгое и всестороннее теоретическое исследование электростатики сфер и цилиндров. — Единый формализм, используемый при решении широкого круга задач. Физики-теоретики и экспериментаторы, химики и инженеры сочтут это бесценным справочником. Он также подходит для продвинутых студентов и аспирантов в области электродинамики. НАУКА / Классический электромагнетизмmbisacshSCIENCE / Общая физикаorg / 10.1063 / 9780735423350

Страница ошибки 404

  • Дом
  • Рекламное объявление
    • № РЕКЛАМЫ: 10/2021 (закрытие 11.05.2021 (до 17:00)
    • Публикации водителей в KPPSC (закрытие 11.11.2021 (до 17:00)
  • Расписание
  • Электронные письма
    • Звонок на собеседовании / Должен быть отклонен / Электронные письма с отклонением
    • Электронные письма по шкале стенографических тестов для младших и старших школьников
  • Результаты теста
    • Результаты теста способностей
    • Результаты стенографического теста
  • Результаты интервью
  • KPPSC Крылья
    • Члены KPPSC
    • Офицеры КППСК
    • Группа ИТ KPPSC
  • Контакты
  • Подать заявку онлайн
Ошибка

Страница не найдена !

Важные ссылки
  • Упреждающее раскрытие информации в соответствии с законом о праве информации, 2013 год
  • Как подать заявление из-за границы
  • Форма заявки
  • Отделения банков
  • Архив объявлений
  • Загрузки
Другие PSC
  • Комиссия Федеральной государственной службы
  • Комиссия государственной службы Пенджаба
  • Комиссия государственной службы провинции Синд
  • Комиссия государственной службы Белуджистана
  • Комиссия государственной службы AJK
  • Возрастная релаксация
  • Заполнение форм
  • Советы при собеседовании
  • Словарь
  • Зоны
  • Ваши комментарии
  • Загрузки
  • Архив объявлений
  • Познакомьтесь с командой
  • Члены PSC
  • Карта сайта
  • Программа PMS
  • PMS старые бумаги
  • Гражданские судьи, старые документы
  • График PMS
  • Tehsildar / Naib Tehsildar Syllabus
  • Упреждающее раскрытие информации в соответствии с Законом о праве информации, 2013 г.

alexxlab

Добавить комментарий

Ваш адрес email не будет опубликован. Обязательные поля помечены *